Você está na página 1de 1437
Bont ERT WIN A CASH AWARD OF Rs. 200.00 It has come to our notice that some booksellers are fraudulently selling fake/duplicate copies of some of our fast selling titles In our sincere efforts to provide you with our genuine books and to protect you against these counterfeit books, Laxmi Publications (LP) has put a Hologram on the cover of some of its fast selling titles. The Hologram displays a unique 3D multi-level, multi-colour effect from different angles. It has the following three levels of flat graphies merged together. The background artwork seems to be ‘under’ or ‘behind’ the Hologram and gives the illusion of depth unlike the fake Hologram on the fake/duplicate books. ci td Channel ‘Channel 2 Background Ovriay Presently, only some titles have got the Holograms. In this case, Comprehensive Physics for Class XII New edition, priced at Rs. 550.00 has got the Hologram. If you or any of your friends finds anywhere in Iadia/abroad any book of this Edition without the LP Hologram, he/she is requested to write to us at M/s LAXMI PUBLICATIONS PVT. LTD., 7/19, Ansari Road, Daryaganj, New Delhi-110002, giving the name and address of the bookseller from where he/she purchased this hook, together with the photocopy ofthis page and the front title cover of the book. He/ She will be sent a cath ‘award of Rs. 200.00. How to decide if the book is genuine or fake ? 1. The above information may or may not be printed. 2. The counterfeit edition of the book may have no LP Hologram or if it has, it will be without the illusionary depth as described above. What is the harm in purchasing duplicate books,? — Poor quality of paper and printing which affect your eyes. — No royalty to authors who are scholars and have put their hard labour in writing the book, thus depriving them of their intellectual rights. Published by : LAXMI PUBLICATIONS (P) LTD 22, Golden House, Daryaganj, New Delhi-110002. _ [011-23 262368 _ { 01423 25 25 72 a +: {ot 38 262870 Fares: { una 96 2970 Branches 129/1, IlIrd Main Road, IX Cross Chamrajpet, Bangalore (Phone : 080-26 61 15 61) 26, Damodaran Street, T. Nagar, Chennai (Phone : 044-24 34 47 26) St. Benedict's Road, Cochin (Phone : 0484-239 70 04) Pan Bazar, Rani Bari, Guwahati (Phones : 0361-254 36 69, 251 38 81) 4-2-453, Ist Floor, Ramkote, Hyderabad (Phone : 040-24 75 02 47) Adda Tanda Chowk, N.D. 365, Jalandhar City (Phone ; 0181-222 12 72) 106/A, Ist Floor, S.N. Banerjee Road, Kolkata (Phones : 033-22 27 37 73, 22 27 52 47) 18, Madan Mohan Malviya Marg, Lucknow (Phone : 0522-220 95 78) 128A, Block 3, First Floor, Noorani Building, LJ. Road, Mumbai (Phone : 022-24 46 39 98) Radha Govind Street, Tharpagna, Ranchi (Phone : 0651-230 77 64) EMAIL : colaxmi@hotmail.com WEBSITE : www laxmipublications.com All Rights Reserved with the Publishers ‘T12-8907-550-COMP. PHYSICS XII | Price : Rs. 550.00 Only ‘Typesetted at : GOSWAMI PRINTERS —_Printed at : AJIT PRINTERS Pr EU CO $$ Chapter Pages Syllabus ““ (viii)x) 1.01. Frictional Electricity and Two Kinds of Charges = 1,02. Conservation of Electric Charges 1.03. Comparison of Charge and Mass 1.04. Quantisation of Electric Charge 1.05. Additive Nature of Charge 1.06. Coulomb's Law—Forces between Two Point Electric Charges 1.07__Coulomb’s Law in Vector Form 1,08. Coulomb's Force between two charges in terms of their Position Vectors = 10 1.09. Units of Charge - 11 1.10. Relative Permittivity or Dielectric Constant - 12 1.11. Comparison of Electrostatic and Gravitational Forces w= 13 1.12. Continuous Charge Distribution os 28 eee a0 1.14. Physical Significance of Electric Field a 31 1.15. Electric Field Strength 31 1.16. Electric Field Due to a Point Charge : 32 1.17. Electric Fields due to Different Charge Distributions = 34 1.18. Electric Field due to a Uniformly Charged Ring at a Point on the Axis of the Ring - 34 1.19. Superposition Principle 35 1.20, Forces between Multiple Electric Charges 37 1.21. Electric Lines of Force [Electric Field Lines] o 45 1.22. Neutral Point 0g 1.23. Electric Dipole, Dipole Field and Electric Dipole Moment = 50 1.24. Electric Field Intensity due to an Electric Dipole at a Point on its Axial Line . 51 1,25. Electric Intensity due to an Electric Dipole at a Point on the Equatorial Line : 52 1,26. Electric Field Intensity at a General Point due to Short Electric Dipole _. 54 1.27. Dipole Field . 55 1,28. Electric Field Intensity at a Point due to two Electric Charges os 55 (ai) Chapter Pages 1.29. Behaviour of Electric Dipole in a Uniform Electric Field = 59 1.30.__Another Definition of Electric Dipole Moment. = 60 1.31. Motion of an Electron in an Electric Field 62 Typical Numerical Problems 65 Very Short Answer Questions 68 Short Answer Questions 70 Conceptual Problems 15 Long Answer Questions we 17 2. ELECTROSTATIC POTENTIAL AND GAUSS'S THEOREM 79—166 2.01. The Mathematics of Line, Surface and Volume Integrals - 79 2.02. An Important Property of Electrostatic Fields 79 2.03__Electrostatic Field—A Conservative Field 2.4\|\|\|.. 2.04. Physical Significance of Line Integral of Electric Field = 82 2.05. Concept of Electrostatic Potential = 82 2.06. Physical Meaning of Potential Difference and Potential 85 2.07. Gold Leaf Electroscope 87 2.08. Units of Electrostatic Potential - 87 2.09. Electric Potential due to a Point Charge ~ 88 2 Po jal at a P due of Charges 9 2.12. Electric Potential at a Point due to Continuous Charge Distributions... 97 2.13. Potential due to a Charged Conductor (Hollow or Solid) = 98 2.14. Coalescence of Charged Drops ~ 100 2.15. Relation Between Potential Gradient and Electric Field Strength - 101 2.16. Potential Gradient (Optional Reading) ~ 103 2.17. Electric Potential due to an Electric Dipole at a Point on the Axial Line of the Electric Di - 105 2.18. Electric Potential due to an Electric Dipole at a Point on the Equatorial Line 105 2.19. Electric Potential at a General Point due to a Dipole 106 2.20. Equipotential Surfaces i 108 2.21. Importance of Equipotential Surfaces Zz 109 2.22. Electrostatic Potential E: ofa: of Two Point ss 11 2.23. Electrostatic Potential Energy of a System of Three Point Charges 111 2.24. Electrostatic Potential Energy of a System of n Charges 113 2.25. Commonly Used Units of Electrostatic Energy = 114 2.26. Work Done in ‘an Electric in a Uniform Electric Field Through an Angle a 114 2.27. Electric Potential Energy of an Electric Dipole in an Electric Field z 115 2.28. Area Vector « 123 2.29. Solid Angle 124 2.30. Electric Flux 124 2.31. Gauss’s Law for Electrostatics or Gauss’s Theorem -~ 126 (xiii) Chapter Pages 2.32. Importance of Gauss’s Law 127 2.33. Proof of Gauss’s Law for Spherically Symmetric Surfaces 128 128 2.34. Proof of Gauss’s Law for a Closed Surface of any Shape 2.35._Deduction of Coulomb's Law from Gauss’s Law 2.0. dD 2.36. Electric Field Intensity due to an Infinitely Long Straight Charged Wire 135 2.37. _Blectric Field Intensity due to Uniformly Charged Infinite Plane Sheet... 137 2.38. Electric Field Intensity due to an Infinite Charged Conducting Plate 138 2.39. Electric Field due to Two Infinite Plane Parallel Sheets of Charge 138 2.40. Electric Field Intensity due to a Uniformly Charged Thin Spherical Shell a 140 2.41. Electric Field Intensity due to a Uniformly Charged Non-Conducting Solid Sphere - 141 Typical Numerical Problesm 143 Very Short Answer Questions as 156 Short Answer Questions 92022 Conceptual Problems 7 162 Long Answer Questions = 165 3.__ CONDUCTORS, CAPACITANCE AND DIELECTRICS 167—240 8.01. Conductors and Insulators, Presence of Free and Bound Charges 167 3.02. General Concept of a Capacitor and Capacitance 168 3.03. Principle of a Capacitor 169 3.04, Uses of Capacitors 170 3.05. Units of Capacitance 170 3.06. Variable Capacitors ms 170 3.07. Capacitance of a Parallel Plate Capacitor with and without Dielectric Medium between the Plates S 172 3.08. Force between the Plates of a Capacitor 173 3.09. Spherical Capacitor (Optional) 175 3.10. Capacitance of an Isolated Spherical Conductor = 176 $.11._ Cylindrical Capacitor (Optional) = 177 3.12. Combination of Capacitors a 179 3.13. Capacitors in Series Combination 179 3.14. Capacitors in Parallel Combination - 180 3.15. Energy Stored in a Charged Capacitor 194 3.16. Energy Stored in Series Combination of Capacitors “ 196 3.17. Energy Stored in Parallel Combination of Capacitors * 196 3.18. Energy Density in Parallel Plate Capacitor ‘ 196 3.19. Redistribution of Charges and Concept of Common Potential 197 3.20. Loss of Energy in Redistribution of Charges 197 3.21. Polar Molecules and their Behaviour in an Electric Field 207 3.22, Non-Polar Molecules and their Behaviour in an Electric Field 208 3.23. Polarisation of a Dielectric Slab 208 3.24, Polarisation Vector—A Measure of Polarisation : 209 (xiv) Chapter Pages 3.25, Relation between P and Surface Density of Induced Charges o 209 3.26. Electric Susceptibility of Dielectric = 210 3.27. Dielectric Strength of a Dielectric - 210 3.28. Effect of Inserting a Dielectric Slab in Parallel Plate Capacitor = 210 3.29. Inserting Dielectric Slab (Slab Thickness < Plate Separation) in Parallel Plate Capacitor - 213 3.30. Effect of Inserting Conducting Slab in Parallel Plate Capacitor - 215 331. The Atmosphere and its Electricity (Optional Reading) = 221 3.32. Van De Graff Generator - 225 3.33. Lightning Conductor - 226 Typical Numerical Problems 226 Very Short Answer Questions 232 Short Answer Questions 233 Conceptual Problems 237 Long Answer Questions 239 UNIT I: COMPETITION SECTION, 241—258 1-1. _ Basic Concepts, Important Terms and Formulae . 241 1-2. Multiple Choice Questions - 242 13. Assertion and Reason Problems (for AIJMS Aspirants) ~- 258 ELECTRIC CURRENT 261—356 4.01. Electric Current - 261 4.03. Carriers of Current (Flow of Electric Charges in a Metallic Conductor) 267 4.04. Conductors, Insulators and Semi-Conductors 268 4.05. Concept of Drift Velocity ite 268 4.06. Relation between Drift Velocity and Electric Field = 269 4.07. - Relation between Current and Drift Velocity 5 2m 4.08. Current Density = 272 4.09. Deduction of Ohm's Law and Expression for Resistance = 276 4.10. Electrical Resistance - 278 4.11. Units of Resistance - 280 4.12. Dimensional Formula of Resistance . 280 4.13. Resistivity (Specific Resistance) 281 4.14. Factors Affecting Resistivity - 281 4.15. Units of Resistivity - 282 4.16. Dimensional Formula of Resistivity = 282 4.17. Relation Between Resistivity, Current Density and Electric Field - 282 4.18. Relation Between V,, P,/ and V ~ 282 4.19. Wide Range of Variation of Resistivities 283 4.20. High Resistivity of Nichrome 283 (xv) Chapter Pages 421 Conductance 283 4.22. Electrical Conductivity 283 4.23. Dimensional Formula of Electrical Conductivity - 284 4.24. Macroscopic Form of Ohm's Law—Relation Between J, o and E ~ 284 4.25. Classification of Materials in Terms of Conductivity - 284 4.26. Mobility om 285 4.21. Mbobilities of Free Electrons and Holes - 285 4.28. Mobility in Terms of Relaxation Time 286 4.29. Electron Mobility and Resistivity 286 4.30. Mobility and Conductivity 286 4.31. Mobility and Current 286 4.32. Resistors 293 4.33. Colour Code for Carbon Resistances 293 4.34. Series Combination of Resistors 296 4.35. Parallel Combination of Resistors ~ 297 26. Distribution of El ci in Twn Parallal Ras 299 4.37. Ohmic Conductors, V-I ‘of Ohm's Law [Non-Linear V-I Characteristics) - 324 4.38._ Temperature—Dependence of Resistance 327 4.39. Variation of Resistivity with Temperature 328 4.40. Thermistor 329 4.41. Superconduetivity (Elementary Idea) aw 931 Typical Numerical Problems 2 336 Very Short Answer Questions 342 Short Answer Questions 346 Conceptual Problems 352 Long Answer Questions - 355 5. SOURCES OF emf AND KIRCHHOFF’S LAWS 357—436 5.01. Electromotive Force = 357 5.02. Potential Difference - 358 5.03. Comparison of emf and Potential Difference es 358 5.04, Internal Resistance of a Cell and Circuit Equation 358 BO. Seimcsianton of tulsa! Beristince ofa Gull ir inst Mad 359 5.07. Cells in Series = 369 5.08. Cells in Parallel 372 5.09. Series-Parallel Combination of Cells or Mixed Grouping of Cells a 376 5.10. Kirchhoff's Laws—Illustration by Simple Applications i 379 5.11. Measurement of Voltages and Resistances ia 398 5.12. Application of Wheatstone Bridge for Temperature Measurements. 399 5.13. Metre Bridge or Slide Wire Bridge—Special Case of Wheatstone Bridge ... 408 5.14. Post-Office Box 413 Chapter Pages 5.15. The Potentiometer ~ 416 5.16. Principle of Potentiometer ~ 417 5.17. Sensitivity of Potentiometer in Measurement of Potential Difference 417 5.18. Comparison of emfs ~ 417 5.19. Determination of Internal Resistance of a Cell ____._._...______.. __419 5.20. Comparison of Resistances - 420 Typical Numerical Problems = 426 Very Short Answer Questions - 428 ‘Short Answer Questions ~ 429 Conceptual Problems 7 433 Long Answer Type Questions 435 6.__THERMAL EFFECT OF CURRENT C7481 6.01. Introduction 437 6.02. Thermal Effect of Current and Joule’s Jaw .________.__437 6.03. Cause of Heating Effect of Current mi 438 6.04. Deduction of Joule’s Law of Heating (from Energy Considerations) - 438 6.05. An Alternative Deduction of Joule’s Law of Heating (Based upon the definition of potential difference) os 439 6.06. Experimental Verification of Joule’s Law = 439 6.07.__Electric Power 440 6.08. Electric Energy a 441 6.09. Some Im it Aspects and tions of Hi Effect of Current. 442 $.10. Energy Transfers in an Electric Circuit 2 444 6.11. Power Rating ofa Resistor 20 6.12 _ Power Output of a Source mi 445 6.13. Power Input to a Source = 446 6.14. Efficiency of a Source of emf z 446 6.15. Maximum Power Theorem 002222 Typical Numerical Problems ~ 466 Very Short Answer Questions = 473 ‘Short Answer Questions 0 AT Conceptual Problems 7 479 1.08. Faraday’s First Law of ectrolysis_ 2 485 1.06. ’s Second Law of Electrol; se 486 ‘L.07._Relation between E and Z 2121122 72.08._Faraday Constant 488 (xvii) Chapter Pages 7.09. Measurement of Current by Electrolysis 7.10. Electrolysis of Water 7.11. Minimum Potential Difference Required for the Electrolysis of Water. 490 1.12. Back emf (or Polarisation Potential) in Water Voltameter ____..__491 1.18. Expression for Electrical Energy Consumed in Decomposition of Water... 491 7.14. Wleetrolysis of Copper Sulphate Solution - 491 7.16. Applications of Electrolysis = 492 7.16. Harmful Effects of Electrolysis a 493 7.17, Bources of emf (Cells) ” 503 7.18. Simple Voltaic Cell = 504 2.19._Daniel Coll 505 ‘L.20._Laclanche Cell 506 1.21. Dry Cell = 507 1L.22._Lend-acid Cell 508 ‘123__Alkali Accumulator 510 2.24, Buel Cell or Gas-Pilled Electricee! 92929292922 225. Solid State Cella 514 2.26 Button Cells 515 Typical Numerical Problems 516 Very Short Answer Questions 518 Short Answer Questions . 520 Conceptual Problems - 524 Long Answer Questions x 526 8.__THERMOELECTRIC EFFECT OF CURRENT «527544 8.01. Thermoelectricity—An Introduction e 527 8.02. Seebeck Effect—Elementary Idea 7 527 803. Seeheck Series 9.40299} BB 8.04. Variation of Seebeck emf with Temperature ; Thermocouple ; 8.05. Thermoelectric Power 530 8.06. t, andi, in terms ofa. and 8 S 530 8.07. ‘Two Laws of Thermoelectricity = 531 8.08. Explanation of Seebeck Effect 7 531 8.09. Measurement of Temperature using a Thermocouple ~ 531 8.10. Peltier Effect—Elementary Idea - 534 8.11. Explanation of Peltier Effect = 534 8.12__Peltier Coefficient of a Jumetion J. 8.13.__Determination of Peltier Coefficient £94949}. CB 8.14. Comparison of Peltier and Joule Effects = 536 8.15. Thermoelectric Refrigerator (An Application of Peltier Effect) = 536 8.16. ‘Thomson Effect—Elementary Idea = 537 8.17. Explanation of Thomson Effect 537 (xviii ) \ Chapter Pages 8.18. Thomson Coefficient a8 Typical Numerical Problem a 541 Very Short Answer Questions iS Short Answer Questions and Conceptual Problems = 543 Long Answer Questions " 543 UNIT Il: COMPETITION SECTION ~ 845-572 I-1. Basic Concepts, Important Terms and Formulae ~ 545 IL2__Additional Information a ISA. Multiple Choice Questions (Current Electricity and Sourcesofem) a I-3B. Multiple Choice Questions (Thermal, Chemical and ThermoclectricEffects) = 8 I1-4, Assertion and Reason Problems (for AIMS Aspirants) 7 871 9.01. Oersted’s Experiment 5 9.02. Concept of Current Element and Magnetic Field = 575 's Law or lace’s Law (Magnetic Field Due to a Current Element) a 576 9.04. Vector Form of Biot-SavartLaw TT 9.05. SL. UnitofMagnetic Field 20 8B 9.06. Magnetic Field Due to Infinitely Long Straight Conductor = 578 9.07. Direction of Field Due toa it Current-Carrying Conductor " 581 9.08. Magnetic Field at the Centre of a Circular Current-Carrying Coil o 587 9.09. Current Loop as a Magnetic Dipole oe 589 9.10. Magnetic Field on the Axis of a Circular Coil 596 9.11, Current Loop and Magnetic Dipole = 598 9.12. Field of Current ing Solenoid—A Qualitative = 600 9.13. ic Induction Due to Current ing Solenoid 3 601 9.14. Lorentz Magnetic Force Fe 604 9.15. Force on a Moving Charge in Uniform Electric and Magnetic Fields se 605 9.16__Units of B 605 9.17. Dimensional Expression of B oo 606 9.18, Force on a Current-Carrying Conductor in a Uniform Magnetic Field. 609 9.19. Forces Between Two Parallel Current-Carrying Conductors = 614 9.20, Experimental Demonstration = 615 9.21. Representation of Field Due to Parallel Currents z 615 9.22. Definition of Ampere jal 616 (xix) Chapter: Pages 9.24. ure Botrsn Tw Nen-peale Curent Elemeni (Opsnnn Reding) 617 9.25. Ampere Balance or Current Balance = 618 9.26. Relative Sizes of Electrical and Magnetic Forces im 618 9.27. is Recess eee ae ~ 623 9.28. Coil or D’ Arsonval Type Galvanometer 630 9.29. Secateurs uacuccmaane 632 9.30, Voltage Sensitivity of a Galvanometer = 632 9.31, Pointer Type or Weston or Pivoted Coil Galvanometer = 633 9.32._Conversion of Galvanometer to Ammeter 9.33. Conversion of Galvanometer to Voltmeter 2... CB 9.34. Ampere’s Circuital Law ri 644 35. Applications of Ampere’s Law a 645 9.36. Field Produced by a Current Along a Circular of Infinite a 646 9.37. Magnetic Field at the Centre of a Straight Solenoid os 648 9.98, Magnetic Induction Due to Toroidal Solenoid (Endless Solenoid) 8 649 9.39. Charged particle Moving in a Uniform Magnetic Field za 651 9.40. Moving Charge in Perpendicular Magnetic and Electric Fields = 653 9.41 Cyclotron ~ 659 9.42, Limitation of Cyclotron = 661 Typical Numerical Problems i 663 Very Short Answer Questions i 680 Conceptual Problems - 691 ‘Long Answer Questions _ 695 40. MAGNETISM 0 CCCCCC‘éGO7—-78.4 10.01. Introduction 2022 T 10.02. Bar Magnet and its Properties 697 10.03. Repulsion—Surest Test of Magnetisation 699 10.04. Atomic Theory of Magnetism 699 10.05. Representation of Uniform Magnetic Field os 700 10.06, Properties of Magnetic Lines of Force os 700 10.07. Bar Magnet as an Equivalent Solenoid = 701 10.08. Flux at a Distance r from a Pole 1 702 10.09. Inverse Square Law of Force in Magnetism = 702 10.10. Magnetic Dipole and Magnetic Dipole Moment. _ 703 10.11. Magnetic Flux Density Due to a Short Bar Magnet (Magnetic Dipole) , at a Point on the Axial Line ~ 704 10.12. Magnetic Flux Density Due to a Short Bar Magnet (Magnetic Dipole) at a Point on the Equatorial Line a 705 10.13. Magnetic Flux Density Due to a Short Bar Magnet (Magnetic Dipole) at any Point a 107 (xx) Chapter Pages 10.14. Gauss‘s Theorem in Magnetism au 708 10.15. ona Di in a Uniform Magnetic Field . 708 10.16. Work Done in Rotating a Magnetic Dipole in a Magnetic Field - m2 10.17. Potential Energy of a Magnetic Dipole in an External Magnetic Field... 113 10.18. Terrestrial Magnetism (Magnetism of the Earth) ~ 716 10.19. Theories of Origin of Earth's Magnetism (Optional Reading) 1 717 10.20. A Spectacular Effect Due to Earth’s Magnetism ws 718 10.21. Some Important Terms Used in Terrestrial Magnetism _ 718 10.22. Declination ag 10.23. Dipor Inclination ws 7119 10.24. Horizontal Component of Earth's Magnetic Field = 119 ‘10.25. Apparent Dip 719 10.26. Magnetic Elements of Earth and Their Variation z= 720 10.27. Combined Field of Bar Magnet and Earth, Neutral Point se ‘724 10.28. The Tangent Law a 729 10.29. Tangent Galvanometer i 730 10.30. Reduction Factor of Tangent Galvanometer (K) 732 10.81. Sensitivity and Accuracy of Tangent Galvanometer (Optional Reading) 732 10.32. Vibration Magnetometer 736 a como of Magnetic Moments of Magnets atte ‘Same Size ie 737 1086. of earth's M Se Field at Two Different oe 4 10.36. of | and Electric Circuits ; A Few Important Terms (Optional Reading) ave. 143 10.37. Magnetic Intensity H oe 743 10.38_Relation Retween B and HOO gg 10.39. Intensity of Magnetisation (or Magnetisation) - 144 10.40. Magnetic Susceptibility 744 10.41, Magnetic Induction 7 745 10.42. Magnetic Permeability _ 7145 10.43. Current Loop as a Magnetic Dipole and its Magnetic Dipole Moment __.. 746 10.44. Magnetic Dipole Moment of a Revolving Electron Diy Chapter 1. 10.53. M1. 2, m3. m4. 11.02, 11.03. 11.04. 11.05. 11.06 11.07, 11.08, 11.09, 11.10, 1.11, 11.13. 11.14. 11.15. 11.16, 11.17 11.18, 11.19. 11.20, 11.21, 11.22, 11.23, 11.24, 11.25. 11.26. Pages . Permanent Magnets 759 Typical Numerical Problems 760 Very Short Answer Questions 766 ‘Short Answer Questions 77 Conceptual Problems 779 Long Answer Questions 782 UNIT II: COMPETITION SECTION 785—828 Basic Concepts, Important Terms and Formulae 785 Additional Information 791 Multiple Choice Questions 300 Assertion and Reason Problems (for AIIMS Aspirants) 827 UNIT IV: ELECTROMAGNETIC INDUCTION AND ALTERNATING CURRENT ELECTROMAGNETIC INDUCTION 831—897 11.01. Introduction - 831 . Magnetic Flux 831 . Positive and Negative Flux 831 . Faraday’s Experiments and Concept of Electromagnetic Induction 832 . Faraday’s Laws of Electromagnetic Induction 834 Lenz's Law 835 . Expression for Induced emf and Induced Current 895 . Expression for Induced Charge 835 |. Induced emf by Changing B 836 |. Induced emf by Changing A 836 |. The Case of a Rectangular Loop being Pulled out of a Uniform Magnetic Field with Uniform Velocity 838 851 . Electromagnetic Induction and Lorentz Force (Optional Reading) : 858 |. Discussion of Faraday’s Laws 7 860 . Eddy Currents 861 . Jumping Disc Experiment 862 . Applications of Eddy Currents 863 . Self-Induction 864 ). Coefficient of Self-Induction or Self-Inductance 865 |. Units of Self-Inductance 866 . Relation Between Henry and Abhenry 866 . Relation Between Henry and Weber 866 . Self-Inductance of a Plane Coil 866 . Self-Inductance of a Solenoid 866 . Experimental Demonstration of Self-Induction 867 . Physical Significance of Self-Inductance 867 (xxii) Chapter Pages 11.27. Energy Associated with Self-Inductance 868 11.28. Non-Induetive Winding 868 11.29. Series Grouping of Coils 868 11.30. Parallel Grouping of Coils 869 11.31. Mutual Induetion 871 11.82. Coefficient of Mutual Induction or Mutual Inductance 872 11.83. Units of Mutual Inductance 872 11.34, Factors on which Mutual Inductance Depends 872 11.35, Mutual Inductance of Two Plane Coils 873 11.36. Reciprocity Theorem of Mutual Inductance 873 11.37. Mutual Inductance of Two Long Solenoids 874 Typical Numerical Problems 879 Very Short, Short, Answer Questions 887 Short Answer Questions 889 Conceptual Problems a92 Long Answer Questions 895 12. TRANSIENT CURRENTS 898—912 12.01. Introduction 898 12.02. Growth of Current in LR Circuit 898 12.03. Time Taken by Current to Grow to its Maximum Value in LR Circuit 900 12.04. Decay of Current in LR Circuit 900 12.05. Time Taken by Current to Decay to Zero in LR Circuit 901 12.08. Charging of Capacitor in CR Circuit 903 12.07. Discharging of Capacitor in CR Circuit 904 Typical Numerical Problems 907 Very Short, Short, Conceptual and Miscellaneous Questions 910 Long Answer Questions 912 13. ALTERNATING CURRENTS 913—1001 13.01. Alternating Currents 913 13.02. Average Value of AC over Half Cycle 914 13.03. Mean or Average Value of Alternating emf 914 13.04. Root Mean Square Value and Peak Value of Alternating Current 915 13.05. Root Mean Square value of Alternating emf 917 13.06. AC Instruments 920 13.07. AC Through a Resistor 921 13.08. Power in a Purely Resistive Circuit 923 13.09. Voltage-Current Relationship for an Ideal Inductor 924 13.10. Two Kinds of Time-Varying Voltages 925 13.11. Current-Voltage Relation for a Capacitor 925 13.12. AC Through an Inductor 925 13.13. Inductive Reactance (X,) 927 13.14, Power in a Purely Inductive Circuit 928 Chapter 14, 13.15. 13.16. 18.17. 13.18. 13.19. 13.20, 13.21, 13.22. 13.23. 13.24. 13.25. 13.26. 13.27. 13.28. 13.29. 13.30. 13.31. 13.32. 13.33. 13.34. 13.35. 13.36, AC Through a Capacitor Capacitive Reactance Power in a Purely Capacitive Circuit AC Through LR Circuit Power in LR Circuit Analysis of LR Circuit by Phasor Diagram AC Through CR Circuit Analysis of CR Circuit by Phasor Diagram AC Through LCR Series Circuit Analysis of LCR Series Cireuit by Phasor Diagram The Impedance Triangle Power in LCR Cireuit Wattles Current or Idle Current Choke Coil LC Oscillations—A Qualitative Study LC Oscillations—A Quantitative Study Resonance and Resonance Circuits ‘The Q-Factor Advantages of Alternating Current over Direct Current Disadvantages of Alternating Current over Direct Current Complex Circuit Analysis (Optional Reading) Conducting Wires, Resistors, Capacitors and Inductors in Real Life (Optional Reading) Typical Numerical Problems Very Short Answer Questions Short Answer Questions Conceptual Problems Long Answer Questions ELECTRICAL MACHINES AND DEVICES 14.01, 14.02, 14.03. ‘Transformer Long Distance Transmission of Electrical Energy Generator or Dynamo (Important Application of Electromagnetic Induetion) AC Generator |. Two Phase AC Generator }. Three Phase AC Generator . Distribution of Three Phase AC . DC Generator DC Motor Starter Induction Motor Very Short and Short Answer Questions Long Answer Questions Pages 932 933 935 937 939 939 944 946 950 953 955 955 962 962 964 966 967 974 977 977 977 984 985 993 994 999 1000 1002—1024 1002 1004 1010 1010 1013 1013 1013 1014 1015 1017 1018 1022 1024 (xxiv) Chapter Pages UNIT IV: COMPETITION SECTION 1025-10384 IV-1. Basic Concepts, Important Terms and Formulae wo 1025 IV-2. Multiple Choice Questions = 1026 IV-3. Assertion and Reason Problems (for AIIMS Aspirants) ~ 1033 15. 16, UNIT V : ELECTROMAGNETIC WAVES ELECTROMAGNETIC WAVES 1037—1063 15.01. Maxwell’s Displacement Current - 1087 15.02. Modified Ampere's Circuital Law or Ampere—Maxwell’s Circuital Law 1087 15.03. Property of Continuity 1038 15.04, Maxwell's Equations 1038 15.05. Electromagnetic Waves 1040 15.06. Transverse Nature of Electromagnetic Waves—Analytical Treatment 1040 15.07. Properties/Characteristics of Electromagnetic Waves 1042 15.08 Brief History of Electromagnetic Waves 1049 15.09. Electromagnetic Spectrum (Including Elementary Facts about uses) 1050 15.10. Earth's Atmosphere and Greenhouse Effect 1053 Very Short Answer Questions 1054 Short Answer Questions 1056 Conceptual Problems 1060 Long Answer Questions = 1063 UNIT V: COMPETITION SECTION 1064—1068 V-1. Basic Concepts, Important Terms and Formulae “ 1064 V-2. Multiple Choice Questions = 1064 PHOTOMETRY 1071—1085 16.01. Ray Optics, as a Limiting Case of Wave Optics ~ 1071 16.02. Sources of Light - 1071 16.03. Photometry - 1072 16.04. Luminous Flux = 1073 16.05. Luminous Intensity of Illuminating Power of a Point = 1074 16,06. Illuminance (Intensity of Illumination at any Point on the Surface) 1975 16.07. Luminance _ 1076 16.08. Reflection Coefficient of a Surface - 1076 16.09. Relationship Between Illuminance E and Luminous Intensity I for a Point Source = 1076 16.10. Lambert's Cosine Law o 1076 16.11. Luminous Efficiency = 1077 Chapter Pages 16.12. Factors on which Illuminance Depends ~~ 1077 16.13. Bunsen’s Grease—Spot Photometer ~ 1078 Very Short Answer Questions - 1083 Short Answer Questions ~ 1083 Conceptual Problems - 1084 Long Answer Questions es 1085 17. REFLECTION OF LIGHT 1086—1105 17.01. Regular Reflection - 1086 17,02. Image of a Point in a Plane Mirror - 1086 17,03. Image of an extended Object in a Plane Mirror 1087 17.04. Rotating Mirror a 1087 17.05. Inclined Mirrors ~ 1088 17.06. Reflection of Light from Spherical Mirrors (Curved Mirrors) 1090 17.07. Focal Length and Radius of Curvature 1091 17.08. Spherical Aberration 1092 17.09. The Mirror Equation or Mirror Formula 1093 17.10. Linear Magnification - 1095 17.11. Applications of Spherical Mirrors - 1097 Very Short Answer Questions ~ 1101 Short Answer Questions “ 1102 Conceptual Problems 1104 Long Answer Questions ~ 1105 18. REFRACTION OF LIGHT 1106—1186 18.01. Refraction of Light ~ 1106 18.02. Indexof Refraction orRefractiveIndex 1107 18.03. Principle of Reversibility of Light s 1108 18.04. Refraction through a Parallel Slab ss 1109 18.05. Expression for Lateral Displacement (or Lateral Shift or Internal Shift) - 110 18.06, Refraction through a Compound Slab zs uu 18.07. Apparent Depth of a Liquid wo 112 18.08. Apparent Depth of a Number of Immiscible Liquids we 112 18.09. Apparent Shift on 1112 18.10. Total Internal Reflection 17 18.11. Relation between Critical Angle and Refractive Index ~ 1118 18.12. Atmospheric Refractions (Application of Total Internal Reflection). 1121 18.13. Totally Reflecting Prisms (Application of Total Internal Reflection) - 1123 18.14. Optical Fibres/Fibre Optics (Application of Total Internal Reflection) 1125 18.15. Sparking of Diamonds ove 1127 18.16. Some Observations Explained by Refraction of Light on 1128 Chapter 19. 18.17. 18.18. 18.19. 18.20, 18.21. 18.22. 18.23. 18.24, 18.25. 18.26. 18.27. 18.28. 18.29. 18.30. 18.31. 18.32. Spherical Refracting Surfaces Refraction at Spherical Surfaces First and Second Principal Focal Lengths of a Spherical Surface ‘The Lens Lens Maker's Formula for Thin Lens Derivation of Lens Maker's Formula~Ab-Initio Method First Principal Focus and First Principal Focal Length Second Principal Focus and Second Principal Focal Length Images Formed by Convex Lens Corresponding to Different Positions of Object Images Formed by Concave Lens Corresponding to Different Positions of Object ‘The Lens Formula/Lens Equation Linear Magnification Power of a Lens Lens Combinations Focal Length of Combination of Two Thin Lenses in Contact Spherical Aberration in Lenses Typical Numerical Problems Very Short Answer Questions Short Answer Questions Conceptual Problems Long Answer Questions DISPERSION 19.01. 19.02. ‘Some Important Terms Sam of Angle of Incidence and Angle of Emergence is Equal to the Sum of Angle of Deviation and Angle of i aristan ef Ange of Devitin wth Agia fcc |. Expression for Refractive Index of Material of Prism . Refraction by a Small-Angled Prism for Small Angle of Incidence Derivation of Formula for Refractive Index of Material of a Prism by Calculus Method . Dispersion . Cause of Dispersion Angular Dispersion ). Dispersive Power . Impure Spectrum . Pure Spectrum . Experimental Arrangement for Obtaining Pure Spectrum . Spectrometer . To Find the Refractive Index of the Material of the Pri . Types of Spectra 1163 5 1154 1155 a 1187 ~ 1158 1165 1165 1167 1169 1178 1179 1184 % 1185 1187—1218 1187 a 1187 - 1188 o 1188 we 1189 1189 1192 1192 1193 1193 1194 1194 1195 (i 1195 a 1196 1197 Chapter 19.17. 19.18. 19.19. 19.21. 19.22. 19.23. (xxvii) . Scattering of Light-Blue Colour of Sky and Reddish Appearance of the Sun at Sun-Rise and Sun-Set Rainbow (A Qualitative Study) Rainbow—An Analytical Treatment Deviation Without Dispersion Dispersion Without Deviation Chromatic Aberration in Lenses Achromatism Typical Numerical Problems ~ Very Short Answer Questions Short Answer Questions Conceptual Problems Long Answer Questions OPTICAL INSTRUMENTS 20.01. 20.02. |. The Human Eye 2. Defects of Vision 20.03. The Photographic Camera 20.04. Simple Microscope 20.05. . The Compound Microscope 20.06. The Astronomical Refracting Telescope 20.07. 20.08. 20.09. 20.10. 20.11. 20.12. 20.13. ‘Terrestrial Telescope Galilean Telescope ‘The Reflecting Telescope Cassegrainian Telescope Newtonian Telescope What is Binocular Vision Prism Binoculuar WAVE OPTICS 21.01. 21.02. 21.08. 21.04. |. Introduction ,. Wavefront . Ripple Tank Method to Study Propagation of Waves in Water |. Huygens’ Construction or Huygens’ Principle of Secondary Wavelets 21.05. Laws of Reflection at a Plane Surface (On the Basis of Huygens’ Principle) 21.06. Laws of Refraction at a Plane Surface (On the Basis of Huygens’ Principle) Pages 1200 1200 1202 1204 1204 1205 1206 1213 1213 1214 1216 1216 1219—1261 1219 1220 1227 1230 1234 1242 1248 1249 1262—1310 1262 1264 1266 1266 1267 1269 (xxviii) 21.07, Behaviour of a Plane Wavefront in a Prism, Lens and Concave Mirror 21.08. Electromagnetic Waves 21.09. Charged Particles and Electromagnetic Waves 21.10. Coherent Sources and their Importance in Interference 21.11. Coherent and Incoherent Additions of Light Beams 21.12. Superposition of Waves (Principle of Superposition) 21.13. Interference of Waves (Analytical Treatment) 21.14. Conditions for Constructive and Destructive Interference 21.15. Interference of Light 21.16. Young's Double Slit Experiment 21.17, Conditions for Sustained Interference 21.18, Formation of Colours in Thin Films 21.19. Fresnel’s Biprism 21.20. Lloyd's Single Mirror Typical Numerical Problems Very Short Answer Questions Short Answer Questions Conceptual Problems Long Answer Questions DIFFRACTION AND POLARISATION OF LIGHT 22.01. Diffraction of Light 22.02. Diffraction due to a Single Slit 22.03. Difference between Interference and Diffraction 22.04. The Diffraction Grating 22.05. Limitations of Optical Instruments 22.08. Fresnel Distance, Ray Optics is a Limiting Case of Wave Optics 22.07. Resolving Power 22.08. Polarisation of Waves 22,09. Malus’ Law 22.10. Plane Polarised Light and Nicol Prism 22.11. Polarisation by Reflection—Brewster's Law 22.12, Polarisation by Scattering 22.13. Rotation of the Plane of Polarisation 22.14. Dichroism 22.15. Polaroids (or Sheet Polarisers) 22.16. Uses of Polaroids 22.17. The Doppler Effect for Light Typical Numerical Problems Very Short Answer Questions Short Answer Questions Conceptual Problems Long Answer Questions Pages 1272 2 1273 = 1274 1274 1275 1276 1277 1279 1280 1281 1283 1290 1294 1296 1297 1300 1302 1305 1309 1311—1351 1311 1311 1316 1319 1320 1321 1322 1324 1326 1327 1929 1331 1332 o 1334 1334 1335 1335 1340 1342 1343 1346 1349 Chapter UNIT VI: COMPETITION SECTION VI-1. Basic Concepts, Important Terms and Formulae VI-2. Multiple Choice Questions VI-3. Assertion and Reason Problems (for AIIMS Aspirants) 1352—1383 1352 1353 1382 S ELECTRONS 23.01. Introduction 23,02. Discharge of Electricity Through Gases at Low Pressures 23.03. Properties of Cathode Rays 23.04. Nature of Cathode Rays 23.05. Thomson's Experiment for Determining e/m of an Electron 23.06. Millikan’s Oil Drop Method 23.07. Positive Rays or Canal Rays 23.08. Origin of Positive Rays 23.09. Properties of Positive Rays Typical Numerical Problems Very Short Answer Questions Short Answer Questions Conceptual Problems Long Answer Questions PHOTOELECTRIC EFFECT 24.01. Photon 24.02. Photoelectric Effect 24.03. Experimental Study of Photoelectric Effect 24.04. Laws of Photoelectric Emission 24.05. Einstein's Photoelectric Equation 24.06, Photoelectric Cell 24.07. Applications of Photoelectricity ‘Typical Numerical Problems Very Short Answer Questions Short Answer Questions Conceptual Problems Long Answer Questions WAVE NATURE OF MATTER 25.01. Dual Nature of Radiation 25.02. Dual Nature of Matter/Matter Waves 25.03. De-Broglie Wave Equation 25.04. Bohr’s Postulate of Momentum of Stationary Non-Radiating Orbits 25.05. De-Broglie Wavelength of an Accelerated Electron 1387—1414 1387 1387 1389 1391 1415—1437 1415 1415 1416 1418 1419 1420 1422 1427 1430 1431 1433 1436 1438—1454 1438 1439 1439 1440 1441 Chapter 25.06. 25.07. 25.08. 25.09. (xx) Davisson and Germer Experiment Thomson's Experiment on Electron Diffraction De-Broglie Waves Associated with Atoms and Molecules Elementary Idea of Electron Microscope (Practical Application of De-Broglie Waves) Typical Numerical Problems Very Short and Short Answer Questions Conceptual Problems Long Answer Questions UNIT VII: COMPETITION SECTION VII-1. ‘VII-2. VII-3. Basic Concepts, Important Terms and Formulae Multiple Choice Questions Assertion and Reason Problems (for AIIMS Aspirants) ATOMS 26.01. 26.02. 26.03. 26.04. 26.05. 26.06. 26.07. 26.08. 26.09. 26.10. 26.11. 26.12. 26.13. 26.14. 26.15. 26.16. 26.17. 26.18. 26.19. UNIT VII: ATOMIC NUCLEUS. Introduction Thomson's Model of Atom Rutherford’s Alpha Scattering Experiment Distance of Closest Approach Rutherford’s Nuclear Model of the Atom Drawbacks of the Rutherford Model Bohr’s Atom Model Bohr’s Theory of Hydrogen Atom Origin of Spectral Lines (Hydrogen Spectrum) Energy Levels of Hydrogen Excitation and Excitation Potential Ionisation and Ionisation Potential Drawbacks of Bohr’s Theory Energy Quantisation X-rays Determination of Wavelength of X-Rays X-ray Spectra Moseley’s Law Absorption of X-Rays Typical Numerical Problems Very Short Answer Questions ‘Short Answer Questions Conceptual Problems Long Answer Questions Pages a 1441 - 1442 1443 ~ 1443 ~ 1449 “~ 1450 ” 1451 1453 1455—1464 ~” 1455 1455, ~ 1464 1467—1511 1467 1467 1467 1470 1472 1472 1473 1473 1477 1479 1481 Chapter (aaxi) 27. NUCLEI, RADIOACTIVITY AND MOLECULES 27.01. 27.02. 27.03. 27.04. 27.05. 27.06. 27.07. 27.08. 27.09. 27.10. 27.11. 27.12, 27.13. 27.14. 27.15. 27.16. 27.17. 27.18. 27.19. 27.20. 27.21, 27.22. 27.23. 27.24. 27.25, 27.26. 27.27. 27.28. 27.29. 27.30. 27.31. 27.32. 27.33. 27.34. 27.35. 27.36. 27.37. 27.38. 27.39. 27.40. 27.41. 27.42. Atomic Masses Bainbridge Mass Spectrograph Composition of Nucleus—Protons and Neutrons Proton-Electron Hypothesis Proton-Neutron Hypothesis Size of Nucleus Nuclear Density Isotopes Isobars Isotones Discovery of Neutrons Properties of Neutrons ‘Thermal Neutrons Mass-Energy Relation Mass Defect Binding Energy Units of Energy Nuclear Forces Nature of Nuclear Forces (Meson Theory) Binding Energy Per Nucleon Binding Energy Curve (Variation of Binding Energy per Nucleon with Mass Member) Radioactivity Properties of a-Rays Properties of B-Rays . Properties of Rays Rutherford and Soddy’s Laws of Radioactive Decay Radioactive Disintegration Constant 2 Half-Life Period Formula for Number of Atoms Left behind after n Half Lives Average or Mean Life of Radioactive Elements Units of Radioactivity Alpha Decay Beta Decay Gamma Decay Radio-Isotopes ‘Nuclear Reaction Important Nuclear Reactions Conservation Laws in Nuclear Reactions Nuclear Fission Chain Reaction ‘Theory of an Atom Bomb Controlled Chain Reaction Pages 1512—1569 1512 1512 1514 1514 1615 1515 1515 1516 1517 1517 1517 1518 1518 1518 1520 1520 1520 1621 1522 1523 1523 1526 1527 1527 1527 1528 1529 1529 1530 1531 1531 1536 1537 1538 1539 1541 1542 1542 1542 1543 1545 1545 Chapter 27.43. 27.44, 27.45. 27.46. 27.47. Nuelear Reactor Apsara Nuclear Fusion Energy Source of Stars and Sun Hydrogen Bomb 27.48. Radiation Hazards 27.49. Nuclear Holocaust Typical Numerical Problems Very Short Answer Questions Short Answer Questions Conceptual Problems Long Anawer Questions UNIT VIII: COMPETITION SECTION VIII-1. Basic Concepts, Important Terms and Formulae VIII-2. Multiple Choice Questions VIII-8. Assertion and Reason Problems (for AIMS Aspirants) Pages 1546 1847 1547 1548 1549 1552 1553 1553 1558 1660 1566 1568 1570—1577 1570 1570 1577 UNIT IX : SOLIDS AND SEMICONDUCTOR DEVICES 28. SOLIDS AND SEMICONDUCTOR DEVICES 28.01, 28.02. 28.03. 28.04. 28.05. 28.06. 28.07. 28.08. 28.09. Crystalline Solid Amorphous Solid Unit Cell Crystal Lattice ‘Types of Crystal System ‘Types of Cubic Crystals Electron Energies Energy Bands in Solids (Qualitative Ideas Only) Difference Between Metals, Insulators and Semiconductors using Band Theory |. Electrons and Holes in Semiconductors . Intrinsic Semiconductor . Doping a Semiconductor . Extrinsic Semiconductors . Carrier Concentration in N-Type Semiconductor . P-Type Semiconductor . Distinction between Extrinsic and Intrinsic Semiconductors Characteristics of a Semiconductor . Electrical Resistivity of Semiconductors . PN Junction Diode ). Forward-Biased PN Junction Diode . Reverse-Biased PN Junction Diode 1581—1640 1581 28.22. 28.23. 28.24. 28.25. 28.26, 28.27. 28.28. 28.29. 28.30. 28.31. 28.32. 28.33. 28.34. 28.35. 28.36. 28.37. 28.38. 28.39. ( xxxiii ) Volt-Ampere Characteristics of PN Junction Diode Relation between V and I for a PN Junction Diode— A Quantitative Study Resistance of a Diode PN Junction Diode as Rectifier Use of PN Junction Diode to Make a Battery Eliminator in a Transistor Radio Limitations of a Diode Different Types of PN Junction Diodes Junetion Transistor Working of PNP Transistor Working of NPN Transistor ‘Transistor as Common Base Amplifier ‘Transistor as Common Emitter Amplifier Relation between a and 6 ‘Transistor Characteristics in Common Base Configuration ‘Transistor Characteristics in Common Emitter Configuration ‘Transistor as an Oscillator Advantages of Semiconductor Devices Disadvantages of Semiconductor Devices Very Short Answer Questions Short Answer Questions Conceptual Problems Long Answer Questions DIGITAL ELECTRONICS 29.01. 29.02. 29.03. 29.04. 29.05. 29.06. 29.07. 29.08. 29.09. 29.10. 29.11. 29.12. 29.13. 29.14. 29.15, 29.16. 29.17. 29.18, 29.19. Introduction Binary Algebra Boolean Operation Binary Decisions The OR Gate OR Gate Realisation Practical Application of OR Gate The AND Gate AND Gate Realisation Practical Application of AND Gate The NOT Gate NOT Gate Realisation ‘The NAND (NOT AND) Gate The NOR (NOT OR) Gate EOR Gate or XOR Gate Combination of Gates Binary Adders Elementary Ideas About Integrated Circuits Integrated Circuits—Additional Information 1599 1600 1601 1601 1603 ae 1603 1603 1606 1607 1608 1609 1612 1616 1616 1618 1620 1621 1621 1629 1630 1635 1637 1641—1667 1641 1641 1645 1645 1645 1646 1646 1647 1648 1649 1649 1650 1651 1652 1653 1653 1653 1656 = 1657 (xxiv) Very Short, Short & Conceptual Questions Long Answer Questions UNIT IX: COMPETITION SECTION 1. x2, Basic Concepts, Important Terms and Formulae Multiple Choice Questions UNIT X: PRINCIPLES OF COMMUNICATION PRINCIPLES OF COMMUNICATION 30.01 30.02. 30.03. 30.04. 30.05. 30.06. 30.07. 30.08. 30.09. 30.10. 30.11. 30.12. 30.13. 30.14. 30.15. 30.16. 30.17. 30.18. 30.19. 30.20. 30.21. 30.22, 30.23. 30.24, 30.25. 30.26. 30.27. 30.28. 30.29. 30.30. 30.31. 30.32. 30.33. Introduction ‘Types of Communication Systems Elementary Idea of Analog and Digital Communication Modulation : An Important Step of Communication Systems Amplitude Modulation (AM) Frequency Modulation Phase Modulation (PM) Pulse Modulation Elementary Idea About Demodulation Production and Detection of Amplitude Modulated Wave Digigal Communication and Quantisation of Message Signal (An Optional Reading) Data Transmission and Retrieval—Fax Modems Modems with Reference to Computers Communication Channels Space Communication Propagation of E.M. Waves in Atmosphere Importance of Radio Waves in Modern Communication Amplitude Modulated Transmission ‘Sky Wave Propagation or Ionospheric Propagation Important Terms Used in the Study of Sky Wave Propagation Space Wave Propagation or Tropospheric Wave Propagation Frequency-Modulated Communication (Height of ‘Transmitting Antenna) Basic Principles of Wireless Radio Communication Why Microwaves are Preferred over other Waves to Beam Signals in a Particular Direction ? Microwaves and Radar Microwaves and Telecommunication Satellites Indian Satellites in Space Satellite Communication Satellite Communication—Additional Information Remote Sensing and its Applications Remote Sensing as an Application of Satellite Communi 1660 1666 1668—1678 1668 1669 1681—1768 1681 1682 1683 1686 1689 1692 1693 1694 1696 1697 1700 1702 1703 1704 1707 1708 1708 1709 1710 171 1714 1717 1719 1720 1721 1721 1722 1722 1723 1727 1730 1731 1733 30.34. Line Communication 1734 30.35. Two Wire Transmission Line * 1734 30.36. A Note on Co-axial Cables * 1737 30.37. Telephone Links 1738 30.38. Optical Communication 1744 30.39. Optical Fibres 1744 30.40. Basic Optical Communication Link 1746 30.41. Optical Sources for Communication Links 1746 30.42. Optical Detectors “ 1747 30.43. Optical Fibres—A Detailed Mathematical and Physical Analysis “ 1747 90.44. Principle of Fabricating Optical Fibres 1749 30.45. Lasers—An Introduction 1750 30.46. Stimulated Absorption 1751 30.47. Spontaneous Emission 1751 30.48. Stimulated Emission = 1752 30.49. Population Inversion 2 1752 30.50. How a Laser Works 1753 30.51. Characteristics of Laser Light 1753 30.52. Applications of Lasers 1754 30.53. Three Components of Laser Devices 17568 30.54. Ruby Laser 1787 30.55. Helium—Neon Gas Laser 1758 30.56. Diode Laser 1759 30.57. Elementary Principles of Light Modulation 1760 Very Short Answer Questions o 1763 Short Answer Questions ~ 1766 Long Answer Questions w= 1768 UNIT X: COMPETITION SECTION 1769-1778 X-1. Basic Concepts, Important Terms and Formulae 1769 X-2, Multiple Choice Questions “ 1769 UNIT XI: UNIVERSE UNIVERSE 1777—1806 LOL Introduction 9a 91.02. Astronomy 1778 31.03. Universe 1778 31.04. Two Windows of the Astronomy 1778 31.05. Optical Astronomy 1778 31.06. Radio 1778 31.07. Tools of Optical Astronomy 1778 31.08 Uses of a Telescope 1780 31.09. Uses of a Spectroscope . 1780 81.10. Requisites of a Good Telescope . 1780 81.11. Radio Telescope = 1780 (xxxvi) 31.12. Difficulties in Radio Astronomy ~ 1781 31.13. Advantages of Radio-Astronomy Over Optical Astronomy : 1781 ‘31.14. Space Probes (Artificial Satellites) os 1782 31.15. Important Constituents of the Universe - 1782 31.16. Properties of the Sun 1782 81.17. Structure of Sun 1783 31.18. Composition of Sun 1783 31.19. Solar Activity 1783 81.20. Source of Energy in the Sun 1785 81.21. Estimation of Surface Temperature of the Sun at 1785 91.22. Binany System 1786 31.23. Estimation of the Surface Temperature of a Planet 1787 31.24, Albedo and its Importance o 1787 31.25. Chemical Composition of the Atmosphere of a Planet a 1788 31.26. Recent Findings about Moon 1788 31.27. Recent Findings about the Planets 4 1788 31.28. Moon a 1788 31.29. Asteroids a 1789 31.30. Comets 1789 31.31, Meteors 1789 31.32. Meteorites 1790 31.83, Distinction Between a Star and a Planet 1790 31.34. Strucutre of a Star and its Chemical Composition 1790 31.35. Constellation 1790 31.36. Magnitude of a Star 1790 31.87. Absolute Magnitude and Absolute Luminosity of a Star 1791 31.38, Red Shift of Star 1791 91.39, Hertz Sprung-Russel Diagram 1792 81.40. Life and Death of a Star 1792 91.41. Degenerate Star 1793 31.42. White Dwarf 1793) 91.43. Pigmy or Neutron Star 1793 81.44, Black Holes 1793 31.45. Milky Way 1783 31.46. Types of Galaxies 1794 31.47. Pulsars 1795 31.48. Hubble's Law 4 1795 31.49. Cosmology os 1795 Very Short, Short and Conceptual Questions 1“ 1802 Long Answer Questions ~ 1805 UNIT XI: COMPETITION SECTION 1807—1809 XL-1. Basic Concepts, Important Terms and Formulae 1807 X12. Multiple Choice Questions 1808 Logarithmic and Other Tables ~ @-twiii) UNIT—I Frictional electricity, charges and their conservation ; Coulomb's law—Forces between two point electric charges, Forces between multiple electric charges » Superposition principle and continuous Electric field and its physical significance, electric field due to a point charge, electric field lines , Electric dipole, electric field due to a dipole and behaviour of dipole in a uniform electric field. Electric potential-physical meaning, potential difference, electric potential due to a point charge, 8 dipole and system of charges ; Equipotential surfaces, Electrical potential energy of a system of ‘two point charges and of electric dipoles in an electrostatic field. Electric flux, .tatement of Gauss's theorem and its applications to find field due to infinitely long straight wire, uniformly charged infinite plane sheet and uniformly charged thin spherical shell Conductors and insulators, presence of free charges and bound charges inside @ conductor ; Dielectrics and electric polarisation, general concept of a capacitor and capacitance, combination cof capacitors in series and in parallel, energy stored in a capacitor, capacitance of a parallel plate capacitor with and without dielectric medium between the plates ; Van de Graff generator. This Unit contains following three chapters : 1. Electrostatic Force and Electrostatic Field 2. Electrostatic Potential and Gauss's Theorem 3. Conductors, Capacitance and Dielectrics yaa UNSOLVED NUMERICALS tant) errant) STORY Charles Augustin de Coulomb (1736-1806) Spent nine years as a military engineer in the West Indies. After the French Revolution, he retired to a small estate to do his scientific research. His experiments on the electrostatic force, to find the ‘quantitative form of the law of repulsion earlier stated by Priestley, were done in 1786-89. This led to Coulomb's Law. He also found the inverse square law of attraction and repulsion among unlike and like magnetic poles ; and worked on friction, wind-mills, elasticity of metal and silk fibres among other things. Electrostatic Force and Electrostatic Field HEEQ “FRICTIONAL ELECTRICITY AND TWO KINDS OF CHARGES (a) Introduction When a glass rod is rubbed with silk or an ebonite rod is rubbed with flannel, these acquire a power to attract light bodies such as small pieces of paper. The agency which gives this attracting power is called ‘electricity’. The bodies which acquire this power are said to be electrified or charged. The electricity produced by friction is called frictional electricity. If the charges in a body do not move, then the frictional electricity is also known as static electricity. The branch of Physics which deals with static electricity is called electrostatics. It has been observed by experi- mentation that when substances listed in column I are rubbed with substances listed in column II, they acquire positive charge while substances listed in column II acquire | Wool, Flannel | Amber, Ebonite, Rubber, Plastic negative charge. Any two charged objects in the same column repel each other. Two charged objects from different columns attract each other. If two bodies from the following list are rubbed, then the body appearing early in the list is positively charged whereas the latter is negatively charged. Far, Glass, Silk, Human body, Cotton, Wood, Sealing wax, Amber, Resin, Sulphur, Rubber, ite. (b) Historical Development The Greeks knew something about electric forces arising from frictional electric charges. This was as early as 6th century B.C. One of the founding fathers of Greek science, Thales of Miletus, observed that ‘amber when rubbed with wool is capable of attracting bits of straw. The ‘science of electricity has its roots in this observation. “Elektron” is the Greek name for amber. This is the origin of the words ‘electric charge’, ‘electric force’, ‘electricity’ and ‘electron’. You should have noted that just after you have combed your dry hair with a plastic comb, the comb acquires the ability to attract small pieces of paper. This is, of course, true only if there is no humidity in the atmosphere. *It is a yellow resinous substance found on the shores of Baltic sea. It is hardened sap of a tree, just similar to a pine tree. 4 COMPREHENSIVE PHYSICS—xiI ‘The ancient Greeks also knew that rocks (lodestone or a magnetite iron ore) from the island of Magnesia in Asia Minor | DaTi,# rubbing, @ body could attract small pieces of iron. The ancient Chinese observed | function Tones electrons that if thin long pieces of certain natural iron ores were suspended | and becomes positively horizontally and rely with astring they wouldorient themselves | charged. A body having roughly in anorth-south direction, It was around 1000 A.D. that er work functio! the Chinese made use of magnetic oompass or navigation purposes. | Peomien nefortnly charged Inhis book, “De Magnete” (on the magnet) published in the year 1600, *William Gilbert gave the first scientific account of the early experiences of electric and magnetic effects, He introduced the name ‘electric for substances like amber which when rubbed with suitable materials attracted light bodies. He also stated in his book that in all experiments on frictional electricity, two kinds of electricity are produced. He stated that while electric charges of the same kind repel each other, those of opposite kinds attract each other. The two kinds of electricity were named as ‘resinous’ and ‘vitreous’. The kind of electricity developed on amber when itis rubbed with wool was called resinous probably because amber is a resin. The type of electricity developed on wool was called vitreous, ‘An American scientist-statesman Benjamin Franklin (1706—1790) introduced a conven- tion according to which the kind of electricity that appears on the glass will be called positive while that on ebonite or amber when rubbed with wool will be called negative. So ‘vitreous’ became positive while ‘resinous’ became negative. This convention has ever since been followed. (c) Explanation of frictional electricity in terms of electron transfer ‘When a glass rod is rubbed with silk cloth, some of the electrons are torn off from the atoms or molecules in the glass due to mechanical friction. This is because the electrons are held less tightly in glass than in silk. The ‘torn off” electrons are transferred to the silk, The excess of electrons on the silk appear as “net negative charge” on the silk while the deficit of electrons in the glass rod makes the glass rod positive by an equal amount. (2) Pith ball pendulum An electroscope is a simple instrument used to detect the presence of electric charge on a body. Pith ball pendulum shown in Fig. 1.1 is the simplest type of electroscope. It consists of a light ball made of wood pith and coated with aluminium. It is suspended by a silk thread. The fact that like charges repel each other and unlike charges attract each other can be easily demonstrated by using pith balls Suppose two pith balls are suspended near each other by fine silk threads. At first they will be attracted to an electrified rubber rod and will cling to it. But, a moment later, they will be repelled by the rubber and will Fig. 1.1 Pith ball also repel each other. Ifthe same experiment is performed with a glass rod pendulum that has been rubbed with silk, we will have the same result. On the other hand, a pith ball that has been in contact with electrified rubber when placed near one that has been in contact with electrified glass is found to be attracted. During rubbing, a body “He was personal physician to Queen Elizabeth'I of England. [ ELECTROSTATIC FORCE AND ELECTROSTATIC FIELD 5 CONSERVATION OF ELECTRIC CHARGES Electric charge (quantity of electricity) is conserved in the same manner as linear momentum, energy, angular momentum ete. The law of conservation of charge is the basis of many equations of electricity and magnetism. According to the law of conservation of electric charge, the algebraic sum of positive and negative charges in an isolated system remains constant. In other words, the net charge of an isolated system remains unaltered. This law, like the laws of conservation of energy and of momentum, is one of the fundamental laws of Physics. There are no exceptions to this law and itis universal. Conservation of charge does not mean that charge cannot be produced or destroyed. It simply means that if ever charge is created or destroyed, it will always be in equal and opposite amounts. As discussed earlier, when two bodies are rubbed, we get simultaneously two unlike charges of equal magnitudes. Following examples illustrate the law of conservation of electric charge : 1, When a glass rod is rubbed with silk, negative charge appears on the silk while an equal amount of positive charge appears on the glass rod. In this way, the net charge on the glass-silk system remains zero both before and after rubbing. 2. The following nuclear reaction depicts the radioactive decay of uranium nucleus. Thorium nucleus and o-particle are the products of the reaction. 2 U*® 5 Th? + Het Total charge before decay = + 92e ; Total charge after decay = + 90e + 2e = + 926 So, the total charge is conserved 3. When a y-ray photon of energy equal to or greater than 1.01 MeV passes close to a nucleus, the electric field of the nucleus would annihilate y-ray photon and create a pair of particles— an electron and a positron*. This phenomenon is called pair production. The total charge before and after remains zero. COMPARISON OF CHARGE AND MASS. Charge Mass Ithas an important rol Tt can be positive, negative or zero. It is always positive. The electric charge is independent of the | According to special theory of relativity, the velocity of the body. mass of a body increases with the increase in velocity of the body. The variation of mass with electromagnetism. | It has an important role in gravitation. velocity of a body is given by m where m is the mass of a body moving with velocity v, my is the rest mass and ¢ is the velocity of light in vacuum. Positron is a parucle whose mans is the same as that of an electron. Positron has a positive charge of the same magnitude as that of the negative cnargéwn an electron, COMPREHENSIVE PHYSICS—xi | ‘The net charge of a system remains conserved in all inertial frames of reference ie., the Mass is not conserved by itself in relativistic processes. net charge of a system is relativisticaily invariant. Charge is quantised. ‘The force between two charges may be attrac- tive or repulsive. Quantisation of mass is yet to be established. The gravitational force between two masses is always attractive. Tt may exist without any net charge. 7 | Tt may not exist without mass. QUANTISATION OF ELECTRIC CHARGE During Benjamin Franklin's time, the electric charge was considered to be a continuous fluid. This idea served many purposes. However, atomic theory of matter showed that fluids them- selves are not continuous but are made up of atoms and molecules. It was in 1897 that J.J. ‘Thomson discovered the electron, It was in 1911 that Millikan successfully showed that charges on tiny oil drops are exact multiples of an elementary charge. Now-a-days, we know from a number of experiments that “electric fluid” is not continuous and all charges occurring in nature are positive or negative integral multiples of certain minimum electric charge. This minimum electric charge is the magnitude of electric charge on an electron. This fundamental charge to which we give the symbol e, has the magnitude 1.60210 x 10 coulomb. So, the charge on an electron is —ewhile the charge on a proton is +e. [The charge on a positron is also + e]. The fact that charges ona proton and on an electron are numerically equal but opposite in sign has been checked experi- mentally to an amazing degree of accuracy namely to one part in 10”. Any charge q, no matter what is its origin, is given by where n = 1, 2,3 qz=tne, When a physical quantity such as electric charge exists in discrete “packets” rather than in continuous amount, it is said to be quantised. This fact is referred to as quantisation of electric charge. When suitably examined on a microscopic scale, most of the properties such as mass, energy, angular momentum etc. are observed to be quantised. The property of quantisation of electric charge has to be kept in mind while dealing with electric and magnetic problems at microscopic level. For many practical purposes or large scale phenomena, we can ignore the quantisation of charge and regard the charge as something continuous. In the discussions of classical electricity and magnetism, the quantisation of charge is completely ignored. This is because the quantum of charge e is so small that “graininess” of electricity does not show up in large scale experiments just as, while breathing, we do not feel that air is made up of atoms. Just as Newtonian mechanics does not correctly describe the behaviour of matter on atomic scale, the classical theory of electromagnetism also fails to describe the behaviour of charge on the atomic scale. ANote on Quarks Recently, the existence of particles of charge te and 2e has been postulated. These particles are called quarks. Inspite of extensive research since 1963, no quark has ever been detected in any experiment. But the existence of quarks is suggested by wealth of indirect evidence. There is something in their basic character that makes their detection difficult. However, if quarks are detected at some future date, it will not alter the well-established property of quantisation of ELECTROSTATIC FORCE AND ELECTROSTATIC FIELD charge. It will merely reduce the ‘quantum of charge’ frome to $e or 4e. We can also look at it in the following way : ‘An upquark charge is always associated with a downquark charge £ and they together follow the law of quantisation. . Additional Information On Oct. 17, 1990, Americans Jerome I. Friedman and Henry W. Kendall and Canadian Richard E. Taylor won the Nobel Prize in Physics for making a breakthrough in man’s understanding of the structure of matter. During experiments at the Massachusetts Institute of Technology and Stanford Linear Accelerator centre in California, the three were able to find traces of quarks, the basic building blocks of matter. These experiments showed definitively that there were smaller nuclear particles than protons and neutrons. ‘The discovery was made when protons and neutrons were illuminated with beams from a giant electron microscope, a 8.2 km long accelerator at Stanford Linear Accelerator centre where Taylor is affiliated. ‘Numerical Examples based on Quantisation of Charge Formula used : q=ne Unit used : qin coulomb ea ‘Standard value: electronic charge =-1.6* 10°C Example 1.01. A polythene piece rubbed with wool is found to have a negative charge of 3.2 x 10-7C. (q Estimate the number of electrons transferred (from which to which). (b) Is there a transfer of mass from wool to polythene? Solution. (a) We know that q=neorn Since the polythene acquires negative charge therefore it is clear that the polythene gains electrons. (b) Mass gained by polythene = 2 x 10!? x 9.1 x 10-* kg = 1.82 x 10°! kg ‘This mass is of course negligibly small. << Example 1.02. Which is bigger, a coulomb or charge on an electron ? How many electronic charges form ‘one coulomb of charge ? (HPBSE 1997, HBSE 2001) Solution. Number of electronic charges in one coulomb, = 0.625 x 10"? b Clearly, one coulomb of charge is bigger than the charge on an electron. 0 i.e., if both q, and q, are +ve or both q, and g, are negative, then the charges repel each other as shown in Fig. 1.3. On the other hand, if¢,9, < 0i.,, ifthe charges are dissimilar, then the charges attract each other as shown in Fig. 1.4. — —- fF, — > «9 s 7 3 s = s Fy ha Fy Fy ta Fy ——— 1» 4 1 5 Fig. 1.3. Vector form of Coulomb's Fig. 1.4. Vector form of Coulomb's law (for q,92>0) law (for g,9 <0) a a You have either reached 2 page thts unevalale fer vowing or reached your ievina tit for his book. a You have either reached 2 page thts unevalale fer vowing or reached your ievina tit for his book. a You have either reached 2 page thts unevalale fer vowing or reached your ievina tit for his book. ELECTROSTATIC FORCE AND ELECTROSTATIC FIELO 13] Water vapour 1.007 | Porcelain | Turpentine 2.23 Germanium | Teflon 21 Titanates 15—12,000 MERI “COMPARISON OF ELECTROSTATIC AND GRAVITATIONAL FORCES In addition to the electrostatic force between two charged bodies, there also exists a gravitational attraction between them. Let us now compare these two forces which are very similar to each other but differ in some fundamental aspects. Points of Similarity G) Both these forces are central forces, i.e., they act along the line joining the centres of two interacting bodies. (ii) Both the forces obey inverse square law of distance. Gii) Both are conservative forces, i.e., the work done by them does not depend upon the path followed. (iv) Both the forces can operate even in vacuum. Points of Dissimilarity (i) While the gravitational forces are always attractive in nature, the electrostatic forces may be attractive or repulsive. (ii) While the gravitational force does not depend upon the medium, the electrostatic force definitely depends upon the medium. (iii) Electrostatic forces are extremely large as compared to gravitational forces. However, they are weak as compared to nuclear forces. The fact that the electrostatic forces are much stronger than the gravitational forces is clearly revealed from the following illustration, where the magnitudes of the two forces have been compared for an electron-proton system which we come across in hydrogen atom, the simplest ofall the atoms, If air be the medium between electron and proton, then according to Coulomb’s law, the -19)2 1_(eXe) 9,499 16x10)? 4ney 7? r where r metre is the separation between the electron and proton. According to Newton’s law of gravitation, the force of gravitational attraction F, between mm, the electron and the proton is given by F, = G—* . r But mass of electron, m, = 9.1 x 10-*! kg and mass of proton, m, = 1.67 x 10° kg -31 21 F, = 66710 x 91x10 x167 x10 ne r on electrostatic force of attraction isgivenby F, = ewton a You have either reached 2 page thts unevalale fer vowing or reached your ievina tit for his book. a You have either reached 2 page thts unevalale fer vowing or reached your ievina tit for his book. a You have either reached 2 page thts unevalale fer vowing or reached your ievina tit for his book. ELECTROSTATIC FORCE AND ELECTROSTATIC FIELD 17 @ Example 1.16, Two point charges Q and q are placed at distances x and 5 respectively from a third charge 4q. All the three charges are on the same straight line. Calculate Q in terms of q such that the net force ong is zero. [CBSE 1998) Solution, Let us first calculate the force F, between Q and q separated by a distance + pew —— 1° Greg G/F " q Q ‘The force F, between 4q and q is given by : x For net force on q to be zero, ae A Po or Q=-4q 4 (3) G @ Example 1.17. (a) Two insulated charged copper spheres A and B have their centres separated by a distance of 0.5 m. What is the mutual force of electrostatic repulsion if the charge on each is 6.5 x 107 C ? The radii of A and B are negligible compared to the distance of separation. (©) What isthe force of repulsion, if (i) each sphere is charged double the above amount, and the distance between them is halved ? (ii) the two spheres are placed in water ? Given ; Dielectric constant of water is 80. [CBSE 1990 C) Solution. (a) q, = 6.5 x 10-7C, q, =6.5 x 107 C,r=0.5 m, F 0 (65 x 10X65 x 107 F=9x 10! 2 BORN NORIO N =1.5x10°N (05y (6) (i) Each of the two charges is doubled and the distance is halved. Clearly, the new force will be sixteen times the original force. i New force = 1.5 x 10x 16N = 0.24N (ii) When spheres are immersed in water, the force will be reduced by a factor of 80. 2 New force = 45% 20" yy 1.875 x104N [: Fe M Example 1.18. In the previous question, suppose the spheres A and B have identical sizes. A third sphere of the same size but uncharged is brought in contact with the first, then brought in contact with the second and finally removed from bath. What is new force of repulsion between A.and B? Solution. Initially, the charge on sphere A is 6.5 x 10-? C. Sphere C has no charge. When A and C are brought in electrostatic contact, charge is shared equally between them because they are of the same size. So, after the contact, each of the two spheres A and C has a charge of 3.25 x 10°7 C. Now, the sphere C (having charge 3.25 x 10-? C) is brought in contact with B having charge of 6.5 x 10-7 C. The charge is now shared equally between B and C. 18 COMPREHENSIVE PHYSICS—XIi 1 1 So charge on each of two spheres B and Cig 325%10 + 65% 10" o, Now, 4% = 3.25 x 10°C, gy = 4.875 x 1077 Cand r= 8.25 x 1077 x 4.875 x 1077 (O5y @ Example 1.19. An infinite number of charges, each equal to 4 uC, are placed along x-axis at x= 1m, 2m, 4m, 8m and soon. Find the total force on a charge of 1 C placed at the origin. [UT 1995) 875 x 10°" C 5m F=9x 10" N=5.7x10°N 4 1 1 1 Solution. Pa9xxaxios| ata ee. | ‘Sum of infinity of GP series ic 4uc auc 4yc a Osim ™ om ST Here a is first term and r is Fig. 18 common ratio, 11 or P96x 109[14 3 65+ on] or P= 36x10? 1 N= 4 196% 10°N=48x101N 4 @ Example 1.20. Two identical balls, each of mass 0.1 x 10 kg, carry identical charges and are suspended by two threads of equal length. At equilibrium, they position themselves as shown in Fig. 1.9. Calculate the charge on either bal Solution. The ball on the left is in equilibrium under the action of following three forces Wessun ‘alie-con- (@ Tension T in the thread (ii) Weight mg of the ball (iii) Coulomb | sider the equilib- repulsion, F. rium of the ballon ‘Tension T can be resolved into two rectangular components —T cos 60° | the right. and T sin 60° For equilibrium, F = T cos 60°, mg = T sin 60° Fr Dividing, ing “8° or P= mgeot 60° 2.9 = mg cot 60° or — gyio = 4negr® mg cot 60° 4mey Fig. 19 ELECTROSTATIC FORCE AND ELECTROSTATIC FIELD 19 Substituting values, we get q? = (04P (0.1107) x88 x 0577 a1 88x 08TF ¢2 = 1x 10°! C? or q=107C=0.1nC ¢ @ Example 1.21. Two tiny spheres, each having mass m kg and charge q coulomb, are suspended from a point by insulating threads each | metre long but negligible mass. When the system is in equilibrium, each string makes an angle 8 with the vertical. Prove that q? = (4 mgl? sin? @ tan @) 4, [PSEB 1995] Solution. Consider the equilibrium of sphere A. Following forces act on the sphere A. (@ Force F of repulsion on A due to B. (di) Weight mg acting vertically downwards. (iii) Tension T in the string towards the point of suspension O. Resolve tension T into two rectangular components : T cos @ and T sin 9. For equilibrium of A, . 1 ¢ Tain O=F= 7 e Aape and Tens O=mg vidi -i_¢, = 2AC = 21 si Dividing, tan @ = 7 (aBt mg i ButAB=2AC = 21 sin 9 -_¢, tan 8 = 4ne (21 sin 0)” mg or q? = (4 mgi? sin® 6 tan 6) dnc, Fig. L11 Mi Example 1.22. Calculate the resultant force on the 10 microcoulomb of charge in Fig. 1.12. Solution. Let F, be the magnitude of the force exerted on 10 uC of charge by 6 uC of charge. (6x10 C0 x10 C) (03 m)* IfF, is the magnitude of the force exerted on 10 uC of charge by 8 uC’ of charge, then Then FF, =9x 10°Nm?C~ orF,=6N - (8x10™ CX10 x 10-6 C) =9x10°Nm?C > (02 m or F,=18N +10 pC 02m é-suc Wig. 112 COMPREHENSIVE PHYSICS—Xil Both the forces F, and F, act at right angles to each other as shown in Fig. 1.13. Their resultant force Fis given by F= VF? +R? [parallelogram law of vectors) or Fe 46? + 18" N= (56+ 904 N = Y360 N=18.97N Also, if@ is the angle which F makes with F, , then tano= 2-38 23 of o= tan (3) = 71.57 ¢ R 6 WW Example 1.28, Find the force on the centre charge in Fig. 1.14. 2 me 4m od qt4* 10°C = g=-$* 10% 76 «10°C Fig. 1.14 Solution. Let F, be the magnitude of the force with which charge g, is attracted towards q,. 1 We know that Pei (Coulomb's law) 6 C5 x 10°F F,=9% 10° mtg? SAOSIN O45 104 N= 0.45 N g, a 2 Fig. 1.15 IfF, is the magnitude of the force with which charge q, is attracted towards q, , then 10" CX6 x 10° C) 4m) Net force F experienced by q, is directed towards q, and is given by F =F, —F, = (0.0450 - 0.0169) N = 0.0281N 4 Note. The signs of the charges are not employed in the use of Coulomb's law because these are implicit in the direction of the forces. I Example 1.24. Two charges are placed on the x-axis, a charge of +3 uC atx =Oand a charge of-5 »C atx =0.40 m [Fig. 1.16]. Where must a third charge q be placed if the force it experiences is to be zero? Solution. In the first place, we should be clear that g must be placed somewhere on the x-axis. Let us now suppose that the third charge q is positive. Ifq is placed between O and A, the two forces experienced by q will be in the same direction and cannot cancel. If it is placed to the right of A, the attractive force exerted by 5 uC of charge’ will always be larger than the repulsive force of + 3 uC of charge’. So there is no scope for the net force to be zero, It is only in the region to the left of O that the net force can be zero. ‘The above discussion is equally valid if q were considered as negative. Let us suppose that the net force on q is zero when it is placed at a distance of r metre to the left of ©. In that case, the force F, exerted by ‘+ 3 .C of charge’ must be equal in magnitude and opposite in direction to the force F, exerted by '- 5 uC of charge’. F,=9x10°N mec? 6X4 = 16.875 x 10-N = 0.0169N ELECTROSTATIC FORCE AND ELECTROSTATIC FIELD 21 Now Fy=F, 1_g(3x10*)_ 1 (5x10) (r+ 04? a5 a5. a eo roa? * FP Gsoae Tg 71887 x=0 Fig. 1.16 r+04 r 04 04 Subtracting 1 from both sides, we get —"=029 or r= 755 m=138m ¢ Note. The negative value is to be rejected because it gives a point between O and A. So, the correct answer is 1.38 m. Mi Example 1.25. Point charges having values +0.1C,+02UC, .01yC 1m +02 ye 0.3 Cand 0.2 uC are placed at the corners A, B, Cand D respectively A B of a square of side one metre. Calculate the magnitude of the force on a charge of + 1 uC placed at the centre of the square. Taking square root and retaining only positive value, we get =1.29 Solution. AC? = AB? + BO?=1+1=20rAC= V2 m ‘ m A0=2Vim=05 fm Also, AO =CO=BO=D0=05 2m Let F, be the force exerted by the charge of ‘+0.1uC’atAonthe DY — 03Sc charge of + 1 uC’ at the centre O of the square. as we (0.1.x 106 CX1x 10% ©) Fig. LIT Then F,=9%10°N mc 02107 Cx 10" ©) 205)" m' 2 12 or Fy = 2210 X0.1x 10" Ny = 0.0018 newton 2x 025 If Fc is the force exerted by charge at C on charge at O, then 8 6 3 p= 9 x 199 SKIT NLA 10) 5p _ 90310" yy 9.9954 205) 2x 025 Both F,, and Fy act in the same direction. The resultant of F, and Fy, F, = (0.0018 + 0.0054) N = 0.0072 N Force exerted by the charge at B on the charge at O, 29x 19° O2% ion 10") 200.5) N=3.6 x 10N =0.0036N COMPREHENSIVE PHYSICS—XII Force exerted by the charge at D on the charge at O, (02x 10 1x 10%) Fy=9x 10° x05} N=0.0036N Both F,, and Fy, act in the same direction. Resultant of F, and Fy, F, = (0.0036 + 0.0036)N = 0.0072 N ‘The angle between F, and F, is clearly 90°. So, the resultant F of F, and F, is given by F = ¥(0.0072)? + (0.0072) N = 0.0072 V2 N= 0.0072 x 1.414N=0.01018N Ml Example 1.26. The charges shown in Fig. 1.18 are stationary. Find the force on 4 uC charge due to the other two. Solution. Let F, be the magnitude of the force on charge + 4 1C +4ye due to charge +2 uC. (4x 106 C2 x 10° ©) = ° 2 * Then, F,=9x 10°Nm?C* oan or F,=18N Again, if, is the magnitude of the force on charge + 4 uC due to charge +3 uC, then 6 6 F,=9x10°N mC? x 4X10 OB x10") aay (02 m) Net horizontal component, Fig. 1.18 F,= Faene 60" —Fy cos 0" or F, = (2.7) (0.5) ~ (1.80.5) = (2.7 - 1.8) 0.5 N = 0.45 N towards left Not vertical component in the vertically upward direction, F, = F, sin 60° + F, sin 60" = (2.7) 0.866) + (1.8) (0.866) = (2.7 + 1.8) 0.866 N = 3.897 N =3.9N (Rounding off to two significant figures.) Net force, F= (045) + (39)? N= 02025 1521 N = J15.4125 N =3.93N=3.9N (Rounding off to two significant figures.) Fig. 1.20 ELECTROSTATIC FORCE AND ELECTROSTATIC FIELD 23 If0 is the angle which F makes with the positive direction of y-axis, then tena = =0.115 or O= tan-!(0.115) =6.58° 1 Example 1.27. A particle of mass m and charge + q is located midway between two fixed charged particles each having a charge + q and at a distance 2! apart, Assuming that the middle charge moves along the line joining the fixed charges, calculate the frequency of oscillation when it is displaced slightly. Solution, Let the charge +q at the mid-point be displaced slightly to the left. The force F, onthe displaced charge+q a due to charge + q at A is given by 1 ¢ +q - +4 Fit Gre d+ 2 }—__ 4, —__ —__4 ‘The force on the displaced charge + ¢ 2 1 ¢ . due to charge +4 8tB, Fy= Gee GF Fig. 1.21 Net force on the displaced charge + q, alt oat FeF,-FiorF= gre, Qa ant @ +a) Z 1 In SHM, acceleration or Pa -—> =z =| is proportional to dmg [C—O + a) displacement and is directed towards the Since mean position. q Acceleration = =~ x m Regmi The motion is clearly simple harmonic. For simple harmonic motion, acceleration = «x (In magnitude) -{,; 2-9 [47 4 = Viegmi®’ “~ 2x 2x \ neoml* 1 Example 1.28. A thin fixed ring of radius 1 metre has a positive charge 1 x 10 coulomb uniformly distributed over it. A particle of mass 0.9 g and having a negative charge of 1 x 10 coulomb is placed on the axis at a distance of 1 cm from the centre of the ring. Show that the motion of the negatively charged particle is approximately simple harmonic. Calculate the time period of oscillations. OT 1982] Solution, r= 1 m,q = 10-* coulomb, m= 0.9.x 10-* kg ,q=~ 10 coulomb, x= Lem = 10-#m If dF is the force of attraction between a charge seg- 1 dah ment dq and charge qo, then dF = 7 weet dF can be resolved into two components—axial component dF cos @ and transverse component dF sin 8. While the transverse components due to different current elements will cancel out, the axial components will be added up. +. Total force on gp due to the whole of charged ring, F=dF cos 0 Fig. 1.22 24 COMPREHENSIVE PHYSICS—XiI or Pe OF + 7) Since relmandz=0.01m, .. ri+ztmr? 1 1 = Fok Here, k= —— Again, F rie or * [ =% Clearly, the force is directly proportional to displacement. Moreover, it is directed towards the centre of the ring. Thus, the motion of the negatively charged particle is simple harmonic. This motion will take place along the axis of the ring. 3 a ane canail [mm on [mx dregr 09x10 x1 ® Time period in SHM, T= 2n y= 2x | TT won a atx 1a omg second 1 Example 1.29. Two similar helium-filled spherical balloons tied 05x — \¢———0.60m———{ 10~* kg weight with strings and each carrying a charge q float in equilibrium. Find: (a) the magnitude of q, assuming that the charge on each balloon acts as if it were concentrated at the centre, (b) the volume of each balloon. [Neglect the weight of the unfilled balloons and assume that density of air = 1.29 kg m~ and the density of helium in the balloons = 0.2 kg m~). (IT 1967} i) im Solution, Step L. 2T cos @= mg 5x10 x98 De ee oder N= 2.568 x 10-9N [me= 96 - (P03 ~0954 m] Step IL (a) For equilibrium, F = T sin ® 2 22.568 x 10-2 x 2 or 9x 10P x Sag = 2.568 x 10-8 2.568 x 0.3 x 10"? x 0.6? 9x 10° 1ox08 ESTE “ 4°" 3x 108 10 q=2x 10 «0.27766 q=0.56x105C or q=0.56uC In this solution, different steps have been clearly mentioned. This would help in quicker understanding of the solution. Step IL. (b) If Vis the volume of the balloon, then U=m'g +Tcos8 gs x10 or Vx129xg=Vx02xg+ S20 € or Vx 109 =2.5x 10% = 2.204 x10" m* ELECTROSTATIC FORCE AND ELECTROSTATIC FIELD 25 3. 4 10. nh. 12. 13. 14. 15. 16, Ww. Calculate the electrostatic force of repulsion between two charges of 1 uC each separated by 1m in vacuum. [Ans. 9 x 10“ N} Force of attraction between two point electric charges placed at a distanced in a medium is F. What distance apart should these be kept in the same medium so that the force between them becomes F Fe (Ans. 2d) ‘Two similar charges are observed to repel each other with a force of 4.5 kg wt when placed 0.03 m apart in air. Calculate the strength of the charges, (Ans. 2.1 uC] A point charge of 3 x 10-* C is 0.12 m distant from a second point charge of - 1.5 x 10-6 C. Calculate the magnitude of the force on each charge. (Halliday/Resnick/Krane) (Ans. 2.8 N] How far apart must two protons be f the electrical repulsive force acting on either one is equal to its weight ? Given : mass of proton = 1.66 x 10-27 kg. (Halliday/Resnick/Krane) (Ans. 0.12 m] Five electrons have been removed from each atom to form ions. Find the electrostatic force between two such ions, when separated by a distance of 4 A in air. {PSEB 2002] (Ans. 3.6 x 10-* N] Coulomb's law for electrical force between two charges and Newton's law for gravitational force between two masses, both have inverse-square dependence on the distance between the charges/ masses. Compare the strength of these forces by determining the ratio of their magnitude for an electron-proton system. tana. 2.4 x 10% he? Check that the ratio is dimensionless, Look up a table of physical constants and deter- Gm, mine the value of this ratio, What does this ratio signify ? [Ans 2.29 x 10, It signifies the ratio of electrostatic and gravitational force between a proton and an electron.] The force between two charged objects is to be left unchanged, even though the charge on one of the objects is halved, keeping the other the same. If¢ is the original separation between charges, then what is the new separation ? [EAMCET 1986] (Ans. d/J/2) (a) Find the value of two equal charges if they repel one another with a force of 0.1 N when placed 0.5 m apart in air. (b) Determine the magnitude of the charges if they were situated in an insulating liquid whose permittivity was ten times that of vacuum. (Ans. (a) 1.67 pC (6) 5.27 WC) A pith ball A of mass 9 x 10°* kg carries a charge of 5 uC. What must be the magnitude and sign of the charge on a pith ball B held 0.02 m directly above the pith ball A such that the pith ball A remains stationary ? (Ans, - 7.84 pC] Acharge of x 10 coulomb has been distributed between two spheres such that they repel each other with a force of 1 newton when their centres are 2 metre apart. Find the distribution. (Ans. 3.8 x 10°C, 1.2 x 10 C) ‘Two identical insulated metal spheres charged with 10 and — 50 units are placed some distance apart. They are made to touch each other and then separated by the same distance. Compare the force between them before and after contact. (Ans. 5: 4] ‘Two equally charged identical metal spheres A and B repel each other with a force 2 x 10 N. Another identical uncharged sphere C is touched to A and then placed at the mid-point between A and B. What is the net electric force on C? [Roorkee 1961} {Ans. 2 x 10-* N directed towards A] Calculate the ratio of the electrostatic and gravitational forces between two electrons separated by a distance r. (Ans. 4.17 x 10% 26 18. 19. 21, 8 ® 31. COMPREHENSIVE PHYSICS—xXII ‘Two oppositely-charged identical metallic spheres placed 0.5 m apart attract each other with a force of 0.108 newton. When they are connected to each other by a copper wire for a short while, they begin to repel each other with a force of 0.036 newton. What were the initial charges on them ? [Ans. 3 yC, F 1 pC] In hydrogen atom, an electron of mass 9.1 x 10~*! kg revolves about a proton in a circular orbit of radius 0.53 A. Calculate the radial acceleration and angular velocity of electron. (Ans. 9 x 10% ms“, 4.12 x 10%!) How far apart the electrons are if the force exerted by one on the other is equal to the weight of the electron ? (Ans. 5.1 m) Charges of + 2 uC, + 3 wC and ~ 8 pC are placed at the vertices of an equilateral triangle of side 0.1m. Calculate the magnitude of the force acting on the — 8 \C charge due to the other two charges. (Ans. 31.2 N] ‘Two free charges + and + 4g are placed ata distance/ apart. Find the magnitude, sign and location of a third charge which makes the system in equilibrium. [Ans. - 49/9, located /3 from + g] ‘Two fixed point charges 4Q and 2Q are separated by a distance x. Where should a third point charge q be placed for it to be in equilibrium ? [PSEB 1999] (Ans. At a distance 0.59x from 4Q] ‘Two negative charges of unit magnitude and a positive charge are placed along a straight line. At what position and for what value of q will the system be in equilibrium ? Check whether it is stable, unstable or neutral equilibrium. UIT 1973] (Ans. Mid-way position, + in magnitude of either charge, unstable equilibrium] A charge q is placed at the centre of the line joining two equal charges Q. Prove that the system of Q three charges will be in equilibrium if @ :- Four point charges of + 10-? C, ~ 10-7 C, 2 x 10° C, and + 2 x 10-7 Care placed respectively atthe corners A, B, C, D of a 0.05 m square. the magnitude of the resultant force on the charge at D. (Ans. 0.2 newton] Four charges + 50 x 10~* coulomb, ~ 12 x 10~* coulomb, + 36 x 10- coulomb and + 90 x 10-* coulomb, are placed respectively at the corners of a rectangle ABCD, AB being equal to 0.05m and BC being 0.12 m. Find the force on the charge at A. [Ans. 4.1 x 10° N, at an angle of nearly 64° with AB] ‘Two tiny spheres, each of mass 10 kg, are suspended from a point by threads, each 0.5 m long. They are equally charged and repel each other to a distance of 0.28 m. What is the charge on each ? Given:g=10ms~. IPSEB 1998 S ; Karnataka 1992] [Ans. 1.6 x 10°C] Two identically charged spheres are suspended by strings of equal lengths. The strings make an angle of 30" with each other. When suspended in a liquid of density 800 kg m~, the angle remains the same. What ia the dielectric constant of the liquid ? The density of the material of the spheres is 1600 kg m™. {Roorkee 1988 ; IT 1976] [Ans. 2] ‘Two similar balls, each of mass m, are hung from silk threads - auc of length i and carry similar charges. Prove that tho separa- fachC__0.8m iS va tion between the two balls is F ] Assume that the : angle which the strings makes with the vertical is very small. 8 {EAMCET 1985] Calculate the force on the + 20 uC of charge in Fig. 1.25. he {Ans. 3.4 N, Nearly 25° with the vertical.) Fig. 1.25 ELECTROSTATIC FORCE AND ELECTROSTATIC FIELD 27 12 P29 10% 20 -N 1 am -1/(_1 am 4neg x? = 4 4neq d? orx=2d 2 4.5 «9.8 =9 x 10° > (aogy 9x10" x16 x16 x10 = 910 x3 x10 x15 x10 . 667 x 91x 91x10" 012x012 9 1 @ 0.108 = 9 x 10° 2 mg = 7 2p: Take m = 1.66 x 107 kg 4reg @ 2 4, = 925% 1.6 « 10°C 0.096 ~9 x 10° x( =) x 28x 10-%C,r=4x 10m & 9x 10° x8 x10" x8x 1 o? 18x10? = (053 x 107)? 9 16 x10- 7 N F F F=9x1 9x10! x98=9 x1 9 10° x 0"? . jee . _. _(Nm°C*)107] 01x01 Given ratio is Ty Fike] kg kel Fp = 2% 10° x24 x 1078 N ie, no units and no dimensions 01x01 pot m2. 1 _ 12 Fs /F,? + Fy? + 2F4Fp cos 60° ary Pa a For net force on qq to be zero, d (4aXa0) i 290 =p SONa ? v2 @ eo d-x? 9x10 For net force on q to be zero, 0.1=9x 10° and 0.1==~— J a OF 10 Os 4. 440, _ HQX4a) 9x 109 x 5X10 9 9x 10°F x98 # p 10? x Gog? 78x10" x 1 4Qq__1 _@2Qq 1 +49 =5x10°C; 1=9x 10° a, a1 ~ % = Vian + a0)" — 4a 500 400 Aa he When C is touched to A, charge would be equally shared. x 10° x 2x 07)? 0.05)" 10° x2 (107? 2(0.05)" N COMPREHENSIVE PHYSICS—XIi 9x10" x (2x 1077)? ~ (0.08) zontal and vertical components of Fp. o? 50x 10°? x 12x 10% ponte x axe" (0.05) 2 =) Fe = 9 « 10? 50430" «8610 9, (0.08)? +12" 9 ) :—— 9.109 80% 10°? x90 x 10 (0.12) mg Net horizontal force, F, = Fy- Fy sin @ . Tcos@=mg, Tsin@=F;F =mg tan0; 0is n+ Foc very small p= (RoR, tmp + tan = sin 8, P= mg sin ® *e 1 /2 Fo ; Consider hori- Fy =9x1 Fp= ge 3 Consider the equilibrium of one sphere. Fae at "2 “7° Here = represents the we mg separation between the two balls O14 Y(0.5)? - (0.14)? |. F,=F, cos 87°, P, =F, + F, sin 37°, F vi x, Inthe firs case, 5 =" = pa (BP +R, tanp= Ee y CONTINUOUS CHARGE DISTRIBUTION We know that the minimum magnitude of non-zero charge is that on a proton or electron or positron. No charge less than this charge can exist in nature. In this sense, we can say that all charges in nature are ultimately point charges. In fact, the simplest kind of charge distribution is an isolated point charge. Any charge which covers a space with dimensions much less than its distance away from an observation point can be considered a point charge. As an example, if there is a net charge on the earth, the earth can be considered a point charge when its electrical effects at the sun are considered. When the finite size of the space occupied by a collection of charges must be considered, we ignore the quantisation of charge. This is because the quantisation of charge is not observable on a macroscopic scale. Asystem of closely spaced charges is said to form a continuous charge distribution. There are three kinds of continuous charge distributions : Linear charge distribution, Sur- face charge distribution and Volume charge distribution. It is useful to consider the density of a charge distribution in the same way as we define the macroscopic density of a solid or liquid, ignoring the quantisation of mass. The term ‘density of charge’ is used in the following three different ways : (i) Line charge density or Linear charge density Ifthe charge were distributed over a straight line or over the circumference of a circle etc., then we use the term ‘line charge density or ‘linear charge density’. ELECTROSTATIC FORCE AND ELECTROSTATIC FIELD 29 Linear charge density is charge per unit length. It denoted by 2 (lambda) or p (mu). It is measured in coulomb/metre. Ifa charge q is uniformly distributed over a line of length J, then the linear charge density is given by For a ring of radius r carrying a total uni- mill, formchargeq, =. However, if the distribution of charge is not uniform, then we talk of ‘linear charge density at a point’. Let us consider a point, on line/, whose position with reference to the origin of the co-ordinate system is given by r’. If Aq is the charge contained in a small line element A/ around the point under consideration, then the linear charge density at the point under consideration is given by Fig. 1.26. Linear charge density Mf or yr) 4 or dq =r yal Total charge on line/, a= fro 1 (ii) Surface density of charge Ifthe charge were spread over some surface of some shape in a continuous manner, we use the term “surface density of charge” or “area density of charge”. Surface charge density is charge per unit area. It is denoted by o (sigma) and is measured in coulomb/metre*. Ifa charge q is uniformly distributed over a surface S, then the surface density of charge is given by However, if the distribution of charge on a surface is not uniform, then we speak of the “surface density of charge at a point”. ("For q spherical shell of Let us consider a point, on surface S, whose position with refer- | radius r carrying a total ence to the co-ordinate system is given by 7’. The surface density uniform charge q on its of charge at this point is denoted by o(7’). If Aq is the charge | surface, = ram contained in a small area element AS around the jv... “der con- sideration, then Pye Mh oe ofr) = 4% =o(7) OOr)= Lt or ot )= SE or dg =atr) dS Total charge on surface S, q = for ds 30 COMPREHENSIVE PHYSICS—xXII Additional Information ” Why edges and corners of a conductor should be rounded off ? What is corona discharge ? In the case of conductors, the surface density of charge varies inversely as the radius of curvature of the surface or directly as the curvature. At « pointed end, curvature is very iarge and the surface density of charge becomes considerable. Consequently, there is a flow of electrons to or from the pointed end of the conductor. As an example, ifthe pointed end is positively charged, then the electrons of the surrounding atoms or molecules are drawn to the point where they neutralise the positive charge. This process of transfer of charge is called corona discharge. Thus, |p. : ‘conductor can bold sufficient charge on it only if itsedgesand | °-1-57. Surface charge density corners are rounded off. (ii) Volume density of charge When charge is spread over a ‘volume’ in a continuous manner, then we use the term ‘volume density of charge’ or ‘volume charge density’. Volume charge density is charge per unit volume. | For a non-conducting solid It is denoted by p (rho). It is measured in coulomb/metre*. | sphere of radius r carrying Ifa charge q is distributed uniformly over avolumeV, | % total uniform charge q, then the volume density of charge is given by p =zt. 3 fo Ifthe volume distribution of charge is not uniform, then we speak of ‘volume charge density at a point’ and denote it by p(r’). If a charge Aq is contained in a small volume element AV around the point under consideration, then Fe tt or pirat Are yay % PP ay or dq=plr) dV Total charge on volume V = [p(r’) dV v Fig. 1.28. Volume charge density ELECTRIC FIELD There is a region of space around a charge or a system of charges within which other charged particles experience electrostatic forces. This region is spoken of as an electric field. Thus, a particle is said to be in an electric field if the particle experiences an electrostatic force. Theoretically speaking, the electric field extends upto infinity but practically the electric field does not show detectable influence beyond a certain limited distance. ELECTROSTATIC FORCE AND ELECTROSTATIC FIELD 31 PHYSICAL SIGNIFICANCE OF ELECTRIC FIELD Coulomb’s law for forces between charges encourages us to think in terms of action-at-a- distance, If field is introduced as an intermediary between the charges, then the interaction between the charges may be represented as : charge == field == charge ‘The first charge sets up an electric field and the second charge interacts with the electric field of the first charge. So the problem of determining the interaction between the charges is reduced to two separate problems : (i) To determine the electric field, established by the first charge, at every point in space. (ii) To calculate the force that the field exerts on the second charge. Additional Information For electrostatics, the concept of electric field is not really necessary but is convenient. Electric field is a characteristic of the system of charges and is independent of the test charge that we place at a point to determine the field. Electric field is a vector field which is defined at every point in space and may vary from point to point. ‘The true significance of the concept of electric field emerges only when we deal with time- dependent electromagnetic phenomena. ‘The greatest speed with which a signal or information can go from one point to another is ¢, the speed of light. Consider the force between two distant charges q, and q, in accelerated motion. The effect of any motion of g, on q, cannot be instantaneous. There is some time delay between the effect {force on q,) and the cause (motion of q,). It is precisely here that the concept of electric field (strictly, electromagnetic field) is very useful. ‘The field picture is this : the accelerated motion of q, produces electromagnetic waves which propagate with the speed of light, reach q, and cause a force on q,. Even though electric and magnetic fields can be detected only by their effects (forces) on charges, they are regarded as physical entities and not merely mathematical constructs. ‘The concept of field was first introduced by Faraday. It is now among the central concepts in physics. ELECTRIC FIELD STRENGTH The electric field strength at a point in an electric field is the electrostatic force per unit positive charge acting on a vanishingly small positive test charge placed at that point. The electric field strength is also called electric field intensity or simply electric field. IFF is the electrostatic force experienced by a test charge q, at a point, then the electric field intensity at that pointis given by AD Since q, has been taken to be positive therefore the direction of E will be the same as that of F. It is also clear from the definition of E that the electrostatic force on a negatively charged a You have either reached 2 page thts unevalale fer vowing or reached your ievina tit for his book. ELECTROSTATIC FORCE AND ELECTROSTATIC FIELD 33 But = is the fore experienced per unit charge and this a Z Fig. 1.29. Electric field due toa point charge is equal to the electric field strength K(r) at the point P. In magnitude Ifqis greater than zero, then the electric field vector (r) points radially outwards at every point. However, if is less than zero, then the electric field vector points radially inwards. The magnitude of the electric vector follows the inverse square law. If (x, y, z) are the co-ordinates of the observation point P, then Peseyed we Pade Sesk Also, r= (22 +y2 429) and r3 = (22 +9? + 22) a ¢ ae The electric field Now, EO) = a etait eb due to a point ‘The three rectangular components of £(r) are as follows : aS q atk gq El) Ten Ga gt aah B= Ge rae Pee, q - BL)" Gaye?” Numerical Example Based on electric field due to a single point charge Se Galina, Formolaused: B= 77 i Unit of E isN C1, ™ Example 1.30, Determine the electric field vector at a distance of 0.50 m from a charge of ~2uC. (Halliday/Resnick/Krane] Solution. g=-2pC=-2x 10°C, r=0.50m =2x10% . $a- 1.25) 5 a= SF 9x10 art =-7.2x10?NC1 ¢ a You have either reached 2 page thts unevalale fer vowing or reached your ievina tit for his book. a You have either reached 2 page thts unevalale fer vowing or reached your ievina tit for his book. a You have either reached 2 page thts unevalale fer vowing or reached your ievina tit for his book. FORCES BETWEEN MULTIPLE ELECTRIC CHARGES Let us now apply the principle of superposition to calculate the electric force experienced by a charge due to several other charges in its neighbourhood. Consider an arrangement of point charges dy, y,43y ~-» dy located at different points, in space, whose positions with respect to the origin of the three dimensional co-ordinate system are Fy,» Tyy-~--s Fy espectively. If Fy(r,) is the total force experienced by charge q, due to all other charges, then according to principle of superposition, FQ) = Fy + Pyy + Py tt Fy Using vector form of Coulomb's law, we at n-n nh F = Fey te aaa ok Eeaus : oh Ii-gP 4% i-me A IRAE am il f-2. fee Re oo Aaa [re SA + 419; 2-3 +944, In-aP In-ak In-nP Similarly, the total force on charge q, due to all other charges is given by a 1 non ah Fila)= Go | tah Staats a 0 Inn In -rf In general, if F,(r,) is the total force on ath charge located at 7, th Gea Here a = 1, 2, 3, f The meaning of the above equation can be compre- hended by referring to Fig. 1.34. Fig. 1.34, Principle of superposition ‘The principle of superposition can also be applied to calculate the total electrostatic force F onachargeq at the position y due to number of point charges and charge distributions. In this case, applying the principle of superposition, we have + fae. a \r-r'P L Mra] BF ‘The above equation makes it clear that the effects of continuous charge distributions can be calculated by using just Coulomb's law and superposition principle. In the above equation, the variable vector r’ has been used in a ‘general sense’. |Force due to Electric Field { due to a number of charges is obtained by principle of superposition. at Units used : een AteNe 3. rin metre (m). 1 Example 1.31. Four charges, each equal toq, are placed at the four verticesof * B a regular pentagon. The distance of each corner from the centre is a. Find the electric field at the centre of the pentagon. Solution. Imagine a charge q at the corner E also. By symmetry, the field at the centre O will be zero. We conclude from here that the field due to charges at A, B, C and D is equal and opposite of the electric field at O due to charge g. -—t@ So, the required field is 7 Jy along OE. << @ Example 1.32. Calculate the magnitude of an electric field which can just balance a deutron of mass 3.2 x 10°7 kg. (PSEB 1999] Solution, Since a deutron consists of a proton and neutron therefore the charge q on deutron is 1.6» 10°C. 3 Fig. 1.35 mg 822107 X98 10-42 1.96107NC# q 16x10 WW Example 1.33. An oil drop of 12 excess electrons is held stationary under a constant electric field of 2.55 x 10 Vm" in Millikan oil drop experiment. The density of oil is 1.26 x 10° kg m™. Estimate the radius of the drop. Given :g=9.81 ms? ande = 1.6 x 10"?C. Solution, n= 12,¢ = 1.6 x 107°C, E=2.55 x 10¢ Vm, p = 1.26 x 10°kg mg =9.81ms%, radius of drop, r=? Inthe given problem, mg=neE or Sw pg =neE Now, qE=mg or E=— va 19 «vs 3ne EB) [312% 18x 10" x 258 x 10 wo {SmeR) [8% 18% 18% 107° 255 x10ET o g 10-7 88x 10-4 s fe} [ 4x 8.14 x 126 x 10° x 981 | MSSSEID sie 8810 ates @ Example 1.34. Two point charges of 2.0 x 10-7 Cand 1.0x 10-7 Care 0.01 m apart in vacuum. What is the magnitude of electric field produced by either charge at the site of the other ? Use the standard value of 1 Sneq” [PSEB 1998) Solution. 9, = 2.0 10-"C, g, = 1.0 10-7C,K=1,r=0.01m Electric field due tog, at the site of ¢, , B= GL ox 100 2021 NC+=18x107NC+ Electric field due tog, atthe siteofq,, = a 30 108 10210" No 20x108NC* < ELECTROSTATIC FORCE AND ELECTROSTATIC FIELD 39 W Example 1.35. Two point charges 9, = 3 uC and gg=-3 »C are located 0.2 m apart in vacuum. (a) What is the electric field at the mid-point O of the line AB joining the two charges ? (b) Ifa negative test charge of magnitude 1.5 x 10°° Cis placed at this point, what is the force experienced by the test charge ? (CBSE 2008} Solution. Electric field at O due to charge at A, Ey =4 or 3x 106 or Ey = 9x 10° ppg NC 1 =2.7 x 10°N C (along OB) . 14a 4 B Electric field at O due to charge at B, Ey = —— ———_1*>+—+ = te rt aan 3 ne OF, qg=-3uC 29100 2210 yy -1 22.7% 108N C-* (along OB) O.1m 0.1m om re Og Oy NetelectricfieldatO, E=E,+E, Fig. 1.36 = 2.7 x 108 +.2.7 x 108 =5.4 x 10°N C+ (along OB) (&) Force on negative test charge at O = qgE = 1.5 x 10 x 5.4 x 10°N = 8.1 x109N ‘This force will be directed along OA. x2m= 3m IfE be the magnitude of the resultant intensity, then E= f(g x 105)? + 45x 108)? NC* or B= faix 10° + 225 x 108 NC = (306 x 10° NC = 17.49 x 10°N C= 1.749 x 10°NC7 Ml Example 1.40. Fig. 1.42 shows four point charges at the corners of a square of side 0,02 m. Find the magnitude and direction of the electric field at the centre (X) of the square if Q = 0.02 4C. [owe =9x109 Nm? c| [ISC Exam. 1998] rep AC = (0.02)? +(0.02)? m or AC= V3 x0.02m +0 04H o.0am + 0.02 HC oor a y 2 Y\|. +29 “a +obsye Om —aoaye Fig. 1.42 Fig. 1.43 aka} a0=2 2 xo.02m= 2 x0.01m Now, BX =CX=DX=AX= V2 x0.01m 9x 10° x 0.04 x 10°F Electric field at X due tocharge at A, Ey= ~~ Don NCA COMPREHENSIVE PHYSICS—XiI It is along XC. 9x 10° x 0.02 x 10°% Blectri field at X due tocharge at, Eo=———9-(ggy8— NC~ Itis along XA. 9x10° x10% Resultant of E, and Eg, Exc =E,-Bo=~ oon? 10.04-0.021NC+ _ 9x 108 x 0.02 © 2x 001x001 9x 10° x 0.04 x 106 Electric field at X due tochargeatD, Ey=———9-gg8 — NC™. Itis along XD. NC+=9x105NC~, Itis directed along XC. 9x 10° x 0.02 x 10° 2x (0017 9x 10° x 10° 2% (0.00? Electric field at X due to charge at B, Ey = NC~ Itis along XB. Resultant ofE)and Ey, — Eyg=Ey-By= Itis along XD. ‘The angle between Exc and Epg is 90°. Netelectricintensity, E= (9 x 10°)* + (@ x 10°)? NC“!= V2 x9x10°NC @ Example 1.41. Three charges, each equal tog, are placed at the three E, corners ofa square of side a. Find the magnitude of electric field at thefourth comer. 10.04 0.02] =9 x 10°NC+ 1 Solution. Ba-Bo= ge zt 1g By ais nc = V8 It acts along BD produced. 1 @ ancy 2a? It also acts along BD produced. IfEis the total field, then E=E,o+By or BaVix7 1a, ~ ay te @” Ante 2c When two vectors of equal q 1 magnitude act at right angles, or aes their resultant is (2 times either vector. Also, the resultant acts mid- or —9 a+ < way between the vectors. Srega’ EXERCISES 92, Analpha particle is situated in an electric field of strength 15 x 10N C~1. Calculate the force acting onit. fAns. 4.8 x 10-4N] 83. Determine the magnitude of an electric field that will balance the weight of an electron. UHSEB 1995) (Ans. 5.57 x 10" N C- (vertically downward)} ELECTROSTATIC FORCE ANO ELECTROSTATIC FIELD 34, 41. A loop of conducting wire of negligible thickness has radius r metre. It carries a total charge q coulomb. Find the electric field at a point X located . at a distance x metre from the centre along the line perpendicular to the x plane containing the loop and passing through its contre (Fig. 1.45). 1 qe [1 sas or | Fig 145 A pendulum bob of mass 80 mg and carrying a charge of 2 x 10-* Cis at restin a horizontal uniform electric field of 20,000 V m-?. Calculate the tension in the thread of the pendulum and the angle it makes with the vertical. (Ans. 8.8 x 10+ N, Nearly 27°] Two point charges of + 16 uC and -9\C are placed 8 em apart in air. Determine the position of the point at which the resultant electric field is zero. (PSEB 1994] (Ans. 0.24 m from --9 pC] Applying the principle of superposition, calculate the magnitude of net electric intensity at the point B in Fig. 1.46, {Ans. 1.15 x 10!°N C~] xo 3m ig 3m 3m . B 4 A a ein eee Seam ee 0.1 a——1 Fig. 1.46 Fig. 1.47 ‘Two point charges q, and q, of 10 C and —_10*C respectively are placed 0.1 m apart. Calculate the electric fields E,, Ey and E, at points A, B and C respectively (Fig. 1.47). PSCERT (New) [Ans. E, = 7.2 x 10*N C~ towards right, E, = 3.2 x 10‘ N C~! towards left, Eq =9 x 10° N C~ towards right) ‘You are required to hold four equal point charges + q in equilibrium at the corners of a square. Determine the sign and the magnitude of the point charge that will do this if placed at the centre of the square. eer 1+ 2v2 4| [Ane negative, “*2 A circular wire loop of radius r carries a total charge Q distributed uniformly over its length. A small length dt of the wire is cut-off. Find the electric field at the centre due to the remaining wire. Qdt Ans. 3", | Calculate the electric field at the centre of a uniformly charged semi-circular are of radius r and linear charge density 4. a] [ane sa Bneor | P=qE=2x 16x 10x 15x 10*N =48x104N mg _ 91x10" x98 16x10" =5.57 x10 NC+ Since mg acts downwards therefore electric force should act upwards. So, the electric field should act vertically downwards. Consider an infinitesimally small length of the wire. Write down the electric field at X. ‘Then, add up the horizontal components. eE=mg,E= Not =2 or x=024m saesa"= -——_—___-.--- 16 pC =9pC —“o08m ace The resultant of E, and E, isin the direction Ey. 10* By = 9x10 BO +9108 1 9x10°x 10 _ 9x10" x10% (0.057 (O15? Eg = 9 x 10° cos 60° + 9 x 107 cos 60" Fy + Fe cos 45° = F sin 45° Fy + Fe sin 45° = F cos 45° If follows from symmetry considerations that the electric field at the centre of the charged circular wire is zero. From here, we may conclude that the electric fields due to the element of length di and the rest of the wire are equal in magnitude and ‘opposite in direction. So, we have merely to determine the electric field due to the charge element of length dl. ELECTROSTATIC FORCE AND ELECTROSTATIC FIELD 45 ELECTRIC LINES OF FORCE [ELECTRIC FIELD LINES] Inorder to understand and appreciate the concept of electric line of force, consider an isolated system of two charges + q and-q separated by a finite distance. Consider a test charge q, placed at a certain point P. It is under the influence of two forces, F'; due to + q and Fz due to~q. The resultant force, as given by parallelogram law of vectors, is F. The test charge moves in the direction of F. When the test charge begins to move in the direction of F, its distance from both + q and—q changes. Consequently, a new resultant force whose magnitude and direction are different from F comes into play. The test charge now begins to move in the direction of the new resultant force and so on. Thus, the test charge moves along a curved line. This curved line is called an electric line of force. it] Fig. 1.48. Curved line of force Fig. 1.49. B at any point on a line of forve An electric line of force is an imaginary straight or curved path along which a unit positive charge is supposed | glectric lines of force to move when free to do so in an electric field. If the electric | donot physically exist field is due to an isolated charge, then the electric lines of force are _| but they represent real straight. However, if the electric field is due to two or more than two | ##wations. charges, then the electric lines of force are curved. The tangent to an electric line of force at any point gives the direction of Eat that point as shown in Fig. 1.49. Thus, the electric lines can give us a broad picture of the pattern of variation of electric field vector Ein a given region of space. Let us now discuss the properties of electric lines of force. 1.In the case of a positively charged body, the electric lines of force are directed away from the body. As shown in Fig. 1.50, the lines of force diverge out from a positively charged body. These lines of force are supposed to end at the negative induced charge on the surroundings. On the other hand, if the body is negatively charged, then the lines of force are directed towards the body. As shown in Fig. 1.51, the lines of force converge at a negative charge. These lines of force are supposed to start from the positive induced charge on the surroun- dings. It is to be noted here that the direction of electric lines of force is the direction in which the positive test charge is urged to move when free to do so in an electric field. nerf Fig. 1.52. (a) Professor subjects himself to an electric potential of ~ 10° volt by touching electrode ofa Van de Graff generator (b) Experiment is repeated by a student with longer hair. Hairs follow the electric lines of force. In Figs. 1.50 and 1.51, the diagrammatic representation of electric field has been done in terms of field lines. Another way of diagrammatic representation of electric field is shown in Figs. 1.53 and 1.54. Here, the field has been represented in terms of field vectors. The length of each arrow represents the magnitude of the electric field at the point. The direction of the Fig. 1.53 Fig. 1.54 (Representation of field by field vectors) a You have either reached 2 page thts unevalale fer vowing or reached your ievina tit for his book. a You have either reached 2 page thts unevalale fer vowing or reached your ievina tit for his book. [ ELECTROSTATIC FORCE AND ELECTROSTATIC FIELD 49 NUMBER OF LINES OF FORCE AND CHARGE (OPTIONAL) IfAN lines of force cross a small area element AA perpendicularly and E is the electric intonsity at the contre ofthe area clement, then SN « B, Ifwe choose, as the constant of proportionality, then 2 = eg ‘The justification for choosing ¢, is as under : Consider the lines of force issuing from a positive point charge q. Consider a spherical surface of arbitrary radius r surrounding this point charge and concentric with it. The magnitude of the electric field intensity at any point on the surface of the sphere is given by lq Ete . tare ja gi 1 4,14 So, the number of lines af fore per unit area is given by €E= 9% Gap 72" aq 2 Total number of lines of force crossing the spherical surface = eB x surface area of spherical surface = egB x dn? = Zt “4 Ane? =q So, if the spacing of the lines of force is so chosen that the number per unit area equals €,E, then the total number of lines issuing from a positive point charge q is exactly equal to q. [E23 NEUTRAL POINT Inorder to understand the concept of neutral point, let us consider two equal and similar point charges separated by acertain distance as shown in Fig. 1.60. Thereis a point N exactly mid-way between the two charges at which ifatest . charge is placed, it will not experience any force. In other N words, the electric field intensity at the point N is zero. This point N is called the neutral point. Is the equilibrium of the test charge placed at Fig. 1.60. Neutral point N stable ? Ifthe test charge placed at N is moved slightly tothe right or leff, it will experience a restoring force which will tend to bring the test charge back toN. On the other hand, if the test charge is moved slightly above or below N, it ‘i would experience an electric force which would take the x 2 ry test charge away from N. So, the test charge does not experience a restoring force in all directions. Thus, the equilibrium of the test charge is unstable. [It may be noted that a stable equilibrium requires restoring force in all iirections} Fig. 1.61. Neutral point a You have either reached 2 page thts unevalale fer vowing or reached your ievina tit for his book. a You have either reached 2 page thts unevalale fer vowing or reached your ievina tit for his book. [= COMPREHENSIVE PHYSICS—XiI aude peg _| +b? -0-p? bal ene " or, (PP B= 2gxBr_; £=2—2gx2r _ “ oS Fe KP E or If the observation point is so far away from the dipole (i. in comparison tor’, then ifr >> 1) that can be neglected B-_127P3 4ne,K r* = 1 Ope For free space, “Grea E acts in the direction of P. Note. Ifthe observation point is taken to the left of origin then the electric field E will be along the direction of x-axis. ELECTRIC INTENSITY DUE TO AN ELECTRIC DIPOLE AT A POINT ON THE EQUATORIAL LINE An equatorial line of the electric dipole is a line perpendicular to the axial line and passing through a point mid-way between charges. Consider an electric dipole consisting of two point charges ‘-q’ and ‘+ q’ separated by distance 2/ as shown in Fig. 1.65. Let the charges ‘- q’ and ‘+ q’ be located at the points A and B respectively. Let P be an observation point on the equatorial line such that its distance from the mid- point O of the electric dipole is r. Let us assume that the medium between the electric dipole and the observation point has dielectric constant K. IfE, is the electric intensity at P due to charge -q’, 14 B= GnegK AP? But AP? = OP? + AO? =r2+22 1 q Ey 4neK r?+/* Fig 1.65. Bata point on equatorial line Bai is represented both in marnitude and direction by Pe. ELECTROSTATIC FORCE AND ELECTROSTATIC FIELD IfE, is the electric intensity at P due to charge ‘+ q’, then 1g Re 4neyK BP? 1 q BP? = OP* + OB* =r? 4/2) Eas Geek 4? * > En is represented in magnitude and direction by PD. 1 q Clearly, BE oR ee Let us resolve E; and Ep into two mutually perpendicular directions. The components of 3 and Ee slong the equatorial line are equal in magnitude and opposite in direction. So, they will cancel each other. But the components perpendicular to the equatorial line get added up because they act in the same direction. So, the magnitude of the resultant intensity E at Pis given by E=E, cos0+E, cos eg L 1.4, bal Bm Fe K +P Ot eK 4? if or B= 7K Pape x 20080 Bet 4 xx + 080 =a 4negK r+? Pe a +2? o pe a or Ee 4neK (+P? 4neK (+2)? where p(=q x 21) is the electric dipole moment of the electric dipole. Ifr >> J, then J? can be neglected in comparison tor2, The electric fleld intensity, due p-—.2 to an electric dipole, at a point 4regK on the equatorial line is along a line parallel to the axis of the » 1 => dipole and is directed opposite Invector form, B= Gog GP to the direction of dipole ‘The negative sign indicates that E is oppositeto p. LP dre, E is represented both in magnitude and direction by PR. K=) Forvacuum, E= It is interesting to note that the dipole field (e« +) decreases much more rapidly as r compared to the field of a point charge (e~3} 7 54 COMPREHENSIVE PHYSICS—xit When the observation point is far far away or when the dipole is very short, then the electric fleld intensity at a point on axial line is double the electric field intensity at a point on the equatorial line. An alternative treatment (Using parallelogram law of vectors) E?=E,?+E,?+2E,E, cos 2 or E? = 2E,? + 2E,? cos 20 or E? = 2E,?(1 + cos 20) = 2E,?(1 + 2 cos? @- 1) or E?=4B,?cos?@ or E=2E,cos® ELECTRIC FIELD INTENSITY AT A GENERAL POINT DUE TO SHORT ELECTRIC DIPOLE Let P be the general point. Note that this point is neither on axial line nor on equatorial line. Consider a short electric dipole of dipole moment p placed in vacuum. Let O be the mid-point of the dipole. Let the line OP make an angle @ with p. Resolving p along OP and perpendicular to OP, we get p cos @ and p sin 6 respectively. Point Pis on the axial line of dipole of dipole moment p cos 6, Let E: be the electric field inten- sity at P due to p cos 8. pain 1_ 2pcose " Then Ey g PS along PA Fig. 1.66. Eat a general point TE r Let E, be the electric field intensity at Pdue top sin 8, 1 psi Then BE, =< 28in® along PB ary IfE is the magnitude of the resultant electric intensity E then 2 1 EPsE?+E = (a, 3] {4 cos? 6 + sin? 6) 1_ 2 Boterastosen?o or Es ane, =) '3 cos* 8+ cos* 8+sin* 6 1 a or E PF ecos*o+1 "im? IfBis the angle which E makes with Ej, then 4neor® tan * Bp cos 2 . p= tas (Lane) In Fig. 1.66, P has been assigned polar co-ordinates. ELECTROSTATIC FORCE AND ELECTROSTATIC FIELD 55 Spectr Cases Case L When P lies on the axial line of the dipole. [Such a position of the point P is called an end-on position.) - 2 2 O° woot BB. Ein, 3 Y8 cos? 0°41 or Em ie, 7 tan B= fen0" 6 n Peo So, the electric field intensity is along the axial line. Case Hl. When P lies on the equatorial line of the dipole. [Such a position of the point P is called a broadside-on position.} 6 = 90°, cos 6 = cos 90° = 0 ae ___ lp E= Grey 7 Y8e0s? 90°41 or Ex Gury 90° tan B = == or B=90 So, the electric field intensity is perpendicular to equatorial line and anti-parallel to the dipole-axis. DIPOLE FIELD The electric field produced by a dipole is called dipole field. ‘The electric field intensity at any point on the equatorial line is one-half the electric field intensity at a point at same distance on the axial line, At any point on the axial line, the dipole field E isin the direction of dipale moment p-At any point on the equatorial line, Eis in a direction opposite to that of p. Atall points other than those on axial and equatorial lines, the field lines will curve towards or away from the charges. MEER ELECTRIC FIELD INTENSITY AT A POINT DUE TO TWO ELECTRIC saa q, and q, situated at points Assdboapenray ie. 1.67). “TO Lal the potion vectors ofA and B with referenee to the origin O be r, and 7, respec- tively. For ‘the sake of convenience, let us assume that both A ¢. 0, F, and 7, lie on the Z-axis of the co-ordinate system such that Aand B are equidistant from O and on opposite sides of O. If a be the distance between two charges, then the coordinates of A and B will be (0, 0, a/2) and (0, 0, —-a/2) respectively. Let us calculate the electric field at a point P(x, y, z). Let the position of P with reference to O be given by r . ‘The electric field at P due to charge q, is given by (0,0. - i. r-r, int, Ir-rye Fig. 1.67. Field due to two charges a You have either reached 2 page thts unevalale fer vowing or reached your ievina tit for his book. a You have either reached 2 page thts unevalale fer vowing or reached your ievina tit for his book. COMPREHENSIVE PHYSICS—XII ® Example 1.44, Two charges, each of 5uC but opposite in sign, are placed 4 cm apart. Calculate the electric field intensity at a point distant 4 cm from the mid-point on the axial line of the dipole. (PSEB 2002] Solution, q=5x10%C, Wa4x10%m, re4x10%m an ops = 29=2)r * Grey (2 - PP BP 9 7 2 2 Es 9x 10" x2x5 x10 xa x4x10 Nc (4x10) - @x 10 144 Sg NC1a10NCI 44 x 10% “ WS Example 1.45. Two point charges of + 0.2 uyiC and -0.2 wyC are separated by 10 m. Determine the electric field at an axial point at @ distance of 0.1 m from their mid-point, Use the standard value of ty, {PSEB 1997] Solution. In the given problem, r >>1. ee 29(22) ‘neq 78 ~ trey 8 12 19-8 or po Melange x10" 10-1 236x104NC4 ¢ @ Example 1.46. Two charges + 10 uC are placed 5 x 10° m apart, #0 Determine the electric field at a point Q, 0.15 m away from O, on a tine passing i through O and normal to the axis of the dipole. i Solution, In the given problem, r >>I fo.Sm 1 p__1 gab E-— 4 dneg 7? ~ 4neq 10x10 x5 x10 10 OT tonc or E=9x« 10 pesos” NC? “BES on O15 x 015 x 015 Fig. 1.68 = L83x 10°N C7 (Ba 43. ‘Two charges « 10 uC are placed 5 x 10-* m apart. Determine the electrie field at a point on the axis of the dipole, 0.15 m away from the mid-point of the dipole and on the side of the positive charge. RENNES (Ans. 2.67 10°N C+) [int Uso = 2-28] 49 44. An electric dipole of length 0.1 m consists of two charges of + 500 uC. Find the electric field due to the dipole at a point on the axis distant 0.2 m from the nearer charge in vacuum. {TAMU 1996} {Ans. 6.25 x 10'NC~)) ELECTROSTATIC FORCE AND ELECTROSTATIC FIELD 59 BEHAVIOUR OF ELECTRIC DIPOLE IN A UNIFORM ELECTRIC FIELD An electric field is said to be uniform if the electric field strength E at every point in the field is the same. A uniform electric field is represented by a set of equal and equispaced straight lines in the same direction. Consider an electric dipole consisting of two charges ‘~q’ and ‘+ q’ placed in a uniform (or homogeneous) external electric field ofintensity E (in the plane of the paper). The length of the electric dipole is 21. The dipole moment P makes an angle @ with the direction of the electric field. ‘Two forces F and— F which are equal in magnitude and opposite in direction act on the dipole as shown in Fig. 1.69. \Fl=|-F| =gE The net force is zero. Since the two forces are equal in magnitude, opposite in direction and act at different points therefore they constitute a couple. Thus, a net torque acts on the dipole about an axis passing through the mid-point of the dipole. veer Fig. 1.69. Dipole in uniform electric field Fig. 1.70. Torque on dipole Now, t= either force x perpendicular distance BC between parallel forces BC But sin = > or BC=2ising Ifthe dipole is in the direction of the t= Fx 2sin@ or t=gEx 2sin® electric field, it will or te(qx2)Esin@ or t=pEsin@ not experience any torque. In vector notation, If both p and E are assumed tobe in the plane of paper, then applying the right hand rule for the cross product of vectors or right-handed screw and cork rule, we find that t is perpendicular tothe plane of the paper and directed inwards (Fig. 1.70). COMPREHENSIVE PHYSICS—Xi! SrEciat Cases Case I. When @ = 0°, then t = pE sin 0° = 0. When the dipole moment p of the dipole is parallel to electric field E , no torque acts on the dipole. In this case, the electric dipole is in stable equilibrium. Case IL. When @ = 90°, then t= pE sin 90° = pE (maximum value) When the dipole moment 7 of the dipole is perpendicular to electric field E, maximum torque acts on the dipole. Case IIT. When @ = 180°, then t = pE sin 180°= 0. When the dipole moment of the electric dipole is anti-parallel to electric field E, notorque acts on the dipole. In this case, the electric dipole is in an unstable equilibrium. ANOTHER DEFINITION OF ELECTRIC DIPOLE MOMENT ‘We know that t = pE sin @ Ifthe electric dipole is held perpendicular to a uniform electric field of unit strength, then E= 1and@= 90°. + P"Txeingo” OT PRT ‘This leads us to the following definition of electric dipole moment. ‘The electric dipole moment of an electric dipole is numerically equal to the torque experienced by the dipole held perpendicular to a uniform electric field of unit strength. Or The electric dipole moment of an electric dipole is numerically equal to the torque required to keep the dipole perpendicular to a uniform electric field of unit strength. Additional Information An electric dipole placed in a homogeneous electric field experiences a torque which tends to align the dipole parallel to the direction of the electric field. If the dipole is free to move, then the dipole shall not come to rest in the equilibrium position. It will overshoot the equilibrium position due to inertia. But, the direction of the torque will also be reversed and the dipole shall again try to come back to the equilibrium position. Thus the dipole shall oscillate and continue to do so for a long time provided there are no dissipative forces. ELECTROSTATIC FORCE AND ELECTROSTATIC FIELD 61 Numerical Examples based on Torque Experienced by Dipole in Electric Field Formulae used: - 1.t=pEsing 2. Tag = PE Units used. LpinCm 2,tinNm Wl Example 1.47. An electric dipole with dipole moment 4 x 10° Cm is aligned at 30° with the direction of a uniform electric field of magnitude § x 10''N C-!, Calculate the magnitude of the torque acting on the dipole. Solution, p =4 x 10°C m, E=5x 10*'NC7,6=30° Magnitude of torque, t=pEsin@ or 124 10%%5 x 10'Nmx 5 =104Nm @ Example 1.48,An electric dipole, when held at 90° with respect to a uniform electric field of 10 NC! experiences a torque of 9 x 10 N m. Calculate the dipole moment of the dipole. (CBSE 1995) Solution, t=pEsin®@ or p= Note that 0is the angle between p os wo and . If 0 is taken as the angle or ee between p and a line normal to “jotxsingso 1/2 =18x10Cm =18x10"Cm < E then t=pE sin (90-0) t=pE cos Example 1.49. *In a certain region of space, electric field is along the Z-direction throughout. The magnitude of the electric field is, however, not constant but increases uniformly along the positive Z-direction at the rate of 10°C! m~!. What ors iva foros eperionend by styler heey hal zal nemo equal to 10-7 Cm in the negative Z-direction ? Solution. In a non-uniform electric field, the force on the electric dipole is given by e® e E Pop, 3 +Py yy tP Oe a ae je T10NC tor mp, =- 107 Cm, p, = 0, p, =0, ae 7% y = F=0+0-10-7 x 105N=-104N The negative sen nae thatthe fr ineted along negative in Letusnow calculate torque. Both p and ff arealong Z-axis. . sin@=0; t=pEsind=0 4 45. An electric dipole consists of two opposite charges, each of 1 uC, separated by 0.02 m. The dipole is placed in an external uniform electric field of 10° N C~. Calculate the maximum torque exerted by the electric field on the dipole. tAns. 0.002 Nm] (int. +,,,,=PE] *Only for the ambitious students. a You have either reached 2 page thts unevalale fer vowing or reached your ievina tit for his book. a You have either reached 2 page thts unevalale fer vowing or reached your ievina tit for his book. a You have either reached 2 page thts unevalale fer vowing or reached your ievina tit for his book. a You have either reached 2 page thts unevalale fer vowing or reached your ievina tit for his book. a You have either reached 2 page thts unevalale fer vowing or reached your ievina tit for his book. a You have either reached 2 page thts unevalale fer vowing or reached your ievina tit for his book. COMPREHENSIVE PHYSICS—xXil 5. Calculate the force on charge q, in Fig. 1.81. Given : g, = - 1.0 x 10-* C, q, = 3.0 x 10-6 C, gy =-2.0 x 10 C, d,, = 0.15 m, dy, = 0.10 mand 8 = 30°, (Halliday Resnick] Fig. 1.81 Hint. Now, use FoR 2+F2 [Ans. 2.6 newton, — 36.6° with x-axis} F,=F,,+F\ssin®, F, = Fy, + F,,0088 Ne law Ut celal Q. 1. What would be special about an object carrying a charge of 5.34 x 10 coulomb ? ‘Ans. It would be a quark of charge 5. Q.2. How is force between two charges affected when dielectric constant of the medium in which they are placed increases? [CBSE 1999] Ans. Since F = e therefore the force decreases. Q.3. The electric charge of macroscopic bodies is actually a surplus or deficit of electrons. Why not protons ? Ans. This is because protons aro tightly bound in the nucleus. So, they cannot be removed easily. Q. 4. Twwo large conducting spheres carrying charges Q, and Gare brought clos to ach thr. the magnitude of electrostatic force between them exactly given by 218s, , where ris the distance between their centres ? ‘Ans. No. Thisis because Q, and Q, shall alter due to the effect of electrostatic induction. Q. 5. Is electric field intensity a scalar or a vector quantity ? Give its SI unit. [CBSE 2003, 1999S) ae Sree CL alg Ans. Electric field inte quantity, Its SI unit is NC~ or Vm. Q.6.A small test charge is released at rest at point in an electrostatic field configuration. Will it travel along the line of force passing through that point ? CER D Ans. Since the initial velocity is zero therefore the test charge shall travel along the line of force. Q. 7. Name the physical quantity whose SI unit is NC~ [CBSE 1998] Ans. Itis SI unit of electric field intensity. Q.8.A charged particle is fired with a velocity 1 making a-certain angle with an electric line of force. Will the charged particle move along the line of force ? ‘Ans. No, the charged particle shall not move along the line of force. Q. 9. What is the absolute permittivity of a medium whose dielectric constant is one ? Ans. e=€K =8.86 x 107? x 1C°N“'m™ or €= 8.86 x 107? CN mm? Q. 10. What is the direction of electric dipole moment vector ofan electric dipole? {CBSE 1992] Ans. It is from negative charge to positive charge. Q. 11. What is the electric field intensity at a point inside a uniformly charged rubber balloon ? Ans. Itiszero. is a vector a You have either reached 2 page thts unevalale fer vowing or reached your ievina tit for his book. a You have either reached 2 page thts unevalale fer vowing or reached your ievina tit for his book. a You have either reached 2 page thts unevalale fer vowing or reached your ievina tit for his book. COMPREHENSIVE PHYSICS—XiI Q. 48. The following figures show wrong representations of the electrostatic fields. Point out the defect in lectrostatic lines of force cannot start from negative charge and cannot end on positive (id) The electrostatic lines of force cannot intersect. (iii) The electrostatic lines of forces cannot form closed loops. Moreover, the electrostatic field lines must be normal to the surface of the charged conductor. Gv) and (v) The electrostatic field lines must be normal to the surface of the charged conductor. o <> SS” wy Fig. 1.84 Q. 49. Ordinary rubber is an insulator. But the special rubber tyres of aircrafts are made slightly conducting. Why is this necessary ? Ans. On account of friction between the tyres and the runway, electricity is produced. Since the rubber tyres of airerafta are marie aight conducting therefore the Trictional electricity i comciucted to the Q. 80. Vehicles carrying inflammable materials usually have metallic ropes touching the ground during motion. Why ? SERRE) | [PSEE 1999; HPSEB 1998) Ans, When a vehicle moves, some frictional electricity is produced due to friction between the body ofthe vehicle and air. Moreover, tie tyres also acquire charge due to friction between the road and the tyres. The metallic ropes conduct charges to the earth. Q.51. Give two properties of electric lines of force. Sketch them for an isolated positive charge. (CBSE 1992] Ans. Refer to Section 1.21. Q.52. Sketch the electric lines of force due to point charges (i) q <0 (ii) q >0. {CBSE 1995] Ans. See Fig. 1.51 and Fig. 1.50. ELECTROSTATIC FORCE AND ELECTROSTATIC FIELD 73 Q.53. An ebonite rod held in hand can be charged by rubbing with flannel but a copper rod cannot be changed like this. Why ? (HPSEB 1997] Ans. Both the human body and copper rod are good conductors of electricity. So, charge flows to the earth. On the other hand, ebonite is a bad conductor. So, the induced charges stay on the ebonite rod. Q. 54. How many electrons will have a total charge of one coulomb ? [PSEB 1999 ; CBSE 1992 ; HPSEB 1992) 1 a Ans. = 4 2—4_,, -6.25x10" ne 16x10 @.86. Calculate the Coulomb force between two protons separated by a distance of 1.6 x 10m. [CBSE 1992] 2 19 19 pn 9X 109 x16 x 10" x16x10 4g gin, @.58. What hind of charges are produced on each when (i) a glass rod is rubbed with sith and (ii) an ebonite rod is rubbed with wool ? ICBSE 1990] Ans. (i) Positive charge will be induced on glass rod and negative charge will be produced on silk. (ii) Negative charge will be produced on ebonite rod and positive charge will be produced on wool. Q. 57. State two properties of an electric charge. (PSEB 1998 8) Ans. (i) Conservation of charge, (ii) Quantisation of charge. Q.58. What is the importance of Coulomb's law in vector form ? UHPSEB 1998 S] Ans. It follows from vector form of Coulomb's law that : @) The forces exerted by two charges on each other are equal in magnitude and opposite in direction. (i) The electrostatic forces are central forces. Q. 69, Calculate the charge carried by 12.5 x 10"® electrons. [CBSE 1992) Ans, Q= 12.5 x 10 «16x 10C=20 Q..60. Calculate the Coulomb force between a proton and an electron separated by 0.8 x 10" m, {CBSE 1992] 9x 10° x 18 x 1079 x 1.6 x 107% Ans Pe Bx 10 x0ax 10% 6x 10°N Q.61. Calculate the Coulomb force between an alpha particle and a proton separated by 5.12 x 10-5 m. {CBSE 1992] 9x 10° x 2 1.6 x10" x16 x 1079 5.12% 5.12 x 107% x 107% Q. 62. Calculate the Coulomb force between 2 a particles separated by 3.2 x 10° m. [CBSE 1992] 9x 10° x2x 1.6 x 10° x2x1.6x 10" 32x 107% x32x 10" Q. 63. The bob of a simple pendulum isa small metal ball of massm. + + + + + + When it oscillates between the two uncharged plates of a parallel plate capacitor, us time period is T. What would be the effect on the time period if the metal bali is given a positive charge and the capacitor is charged as shown in Fig. 1.85? | Ans. Fs N=176N Ans. Fe N=90N Ee Ans. =mg+qB or g=, sat vn = —__"*_ 74 COMPREHENSIVE PHYSICS—xXil Also, T=2n T’ R T= ox fe ios: Clearly, Tot Tl So, the time period is increased. Fig. 1.86 Q. 85. In the previous question, if the capacitor plates are repositioned as shown in Fig. 1.87, then what will be the time period # Ans. mg’ = (mg? + (Qe? . or ge e+(@8) ’ @ |- i ate (#) % wor m Fig. 1.87 Q. 66. Two small balls having equal positive charge q coulomb are suspended by two insulating strings of equal length ! metre from a hook fixed to.a stand. The whole set up is taken in.a satellite into space where there is no gravity. What is the angle between the two strings and the tension in each string ? [HIT 1986] Ans. Since it is a condition of weightlessness therefore weight has no role to play. The strings would become horizontal due to electrostatic force of repulsion. So, angle between the strings is 180°. 2 ‘Tension in each string = = aGN Q.67. What is an electric line of force ? What is its importance ? IPSEB 1998 S] Ans. An electric line of force is an imaginary straight or curved path along which a unit positive charge is supposed to move when free to do s0. ‘The tangent ata point on an electric line of force gives the direction of the electric field at: that point. ‘The relative closeness of electric lines of force in a certain region provides us an estimate of the electric field strength in that region. Q. 68. A copper sphere of mass 2 g contains nearly 2 x 10 atoms. The charge on the nucleus of each atom is 29e. What fraction of the electrons must be removed from the sphere to give it a charge of +2 WC? ‘Ans, Total number of electrons = 292% 10 6 Electrons removed = Sern = 125 x 103 Ratio is 2.16 x 10", Q.69.A sensitive instrument is to be shielded from the strong electrostatic fields in its environment. Suggest a possible way. NCERT (New) Ans. The instrument can be shielded by enclosing it inside a metallic surface. Since no electrostati field can be present within the metallic surface therefore the instrument will be shielded from electric influence. This process is known as electrostatic shielding. ELECTROSTATIC FORCE AND ELECTROSTATIC FIELD . 75 Q.70. Explain two field lines never cross each other at any point. SERRE) (PSEB 2002, 2000, 1999 ; HPSEB 1996 ; CBSE 1992] Ans. The electric lines of force never cross each other, because if they do so, the electric field should have two directions at the point of intersection which is impossible. CONCEPTUAL PROBLEMS Q.71. A block of mass m carrying charge q is placed on a 8 frictionless horizontal surface. The block is connected toa rigid wall k through an unstressed spring of spring constant k. A horizontal uniform electric field E parallel to the spring is switched on. Find the amplitude of the resulting simple harmonic motion of the block. Ans. At the extreme position, the elastic restoring force is ‘equal and opposite to the electrostatic force. Fig. 1.88 ha=qE or a= “ Q.72. Electrons are held in an atom by an electrostatic force of attraction to the nucleus. Protons and neutrons in the nucleus are held together by the nuclear force. Is it a sensible question to ask what holds the electrons (or the protons or neutrons) together ? Ans. We do not yet know what holds electrons together. The forces responsible are sometimes called ‘Poincare stresses’, but that is only to hide our ignorance. Q.73.A charged metallic sphere A is suspended by a nylon thread. Another charged metallic sphere B carried by an insulating handle is brought close to A to.a distance of 9.0m between their centres. The resulting repulsion of A is noted (for example by shinning a beam of light and measuring the deflection of its shadow on ‘calibrated screen). Spheres A and Bare touched by uncharged spheres C and D respectively. Cand Dare then removed, and B is brought closer to A to a distance of 4.5 cm between their centres. What is the expected repulsion of A on the basis of Coulomb's law ? Spheres A and C and spheres B and D have identical sizes. Ignore the sizes of A and B in comparison to the separation between their centres. Ans. If q and q’ be the original charges on A and B respectively, then F = k “4, where ris the separation between A and B. When C touches A, by symmetry, the charge on A is $- Similarly when B touches D, the charge on Bis - ‘The separation between A and Bis also halved. Now, the force is given by (g/2)(@R) _, aa" crap ke =P So, the electrostatic force of repulsion is unchanged. Q. 74. Two identical metallic spheres of exactly equal masses are taken. One is given a positive charge coulomb and the other an equal negative charge by friction. Are their masses after charging equal ? (OT 1983) ‘Ans. No. The positively charged sphere has lost electrons. So, its mass is decreased. On the other hand, the negatively charged sphere has gained electrons. So, its mass is increased. Q. 75. The electrostatic force between two protons situated at a distance x from each other is y newton. What will be the electrostatic force between two electrons situated at the same distance ? Ans. The electrostatic force will remain the same. This is because neither the magnitude of charge nor the distance between the two charges has been changed. Pek 76 COMPREHENSIVE PHYSICS—XIl Q.76.A charge placed at a certain distance on the axial line of an electric dipole experiences a force of 16 newton. What will be the force on the charge if the distance of the charge is doubled ? ‘Ans. In the given problem, F « 4. 7 So, on doubling the distance, the force will be reduced by a factor of 2, -e.,8. Thus, the new force will be 8 N,ie,2N. Q.77.A boy brings the palm of his hand near the disc of a charged gold leaf electroscope. The leaves of the electroscope are observed to collapse slightly. But when the boy moves his hand away from the gold leaf electroscope, the leaves resume their original position. How do you explain the behaviour of the leaves ? Ans. While an equal and opposite charge will be induced on the lower side of the palm, equal and same charge will be induced on the upper side of the palm. But this will leak to the earth. The induced ‘opposite charge causes slight collapse of the leaves. Q.78. Why can one ignore quantisation of electric charge when dealing with macroscopic i.e., large scale charges ? TY ‘Ans. The charge on an electron is extremely small. If a few electrons are added or removed, the charge on the charged body changes by only a very very small amount. So, it appears to vary in a continuous manner. Q.79. Can you suggest some sort of “remote control” method oe for charging a conductor, i.e., the conductor is neither to be rubbed 3 nor to be brought into contact with a charged body ? Ans. When a positively charged glass rod G is brought near an uncharged insulated conductor AB, the conductor AB is charged as shown in Fig. 1.89. The phenomenon of charging a conductor by bringing a charged body near it is called electrostatic induction. This electrification by influence is due to the redistribution of free electrons in the conductor. The phenomenon Fig. 1.89 of electrostatic induction was first discovered by Gray in 1729. Q.80. How can you charge an ‘uncharged insulated conductor’ negatively by electrostatic induction ? Ans. First of all, bring a positively charged glass rod near the given conductor AB. The end A of the conductor will be charged negatively while the end B will be charged positively as shown in Fig. 1.90. Keeping the glass rod near the conductor, the conductor is earthed as shown in Fig. 1.91. Finally, the glass rod and earth-connection are removed. ‘The conductor AB acquires a negative charge as shown in Fig. 1.92. Coreen) B A E B a Fig. 1.90 Fig. 191 Fig. 1.92 G Q.81. It is now believed that protons and neutrons (which constitute nuclei of ordinary matter) are themselves built out of more elementary units called quarks. A proton and a neutron consist of three quarks cach. Two types of quarks, the so called ‘up’ quark (denoted by u) of charge (+ 2/3) e, and the ‘down’ quark (denoted by d) of charge (~ 1/3)¢, together with electrons build up ordinary matter. (Quarks of other types have also been found which give rise to different unusual varieties of matter.) Suggest a possible quark composition ofa (i) proton (ii) neutron. Ni Ans. (i)uud, (ii) udd. ANNAN TUM a You have either reached 2 page thts unevalale fer vowing or reached your ievina tit for his book. a You have either reached 2 page thts unevalale fer vowing or reached your ievina tit for his book. a You have either reached 2 page thts unevalale fer vowing or reached your ievina tit for his book. 80 COMPREHENSIVE PHYSICS—Xil conservative nature of electrostatic field. In order to prove this, consider the electrostatic field of a single stationary point charge. For the sake of convenience, let the charge +q be assumed to be located at the origin O of the three- dimensional rectangular coordinate system. Consider two points A and B in the electrostatic field of charge +g. The positions of A and B with reference tothe origin O are given by position vectors r, and rs respectively. Connect the points A and B by an arbitrarily chosen curve L. It is understood that the points A, B and the curve L avoid the location of source charge (+9) ig. 2.2. Line integral of electric field because there the field would become infinitely large. By Toevaluatethelineintegral |, di, letus divide ~ the curve L into a large number of small segments. One such small segment is CD (= A7). The Positions of end points C and D of the segment with reference to the origin O are given by position vectors r and r’ respectively as shown in Fig. 2.2. = ep Se l1q.- 1 qr > Now, E(r).Al te, 7? # Al inn * 7 Al 2 lq 4 > Gey ord Since the dot product of two equal vectors is equal to the square of the magnitude of either of the two vectors, erie Applying the triangle law of vector addition, we get -’ = r + Al. rPa(p+Al).(r+Al) or rPar ip +r Al +Alr + Al AL or rar427.A0 +42 [ 7.AT = Al 7, dot product being commutative} ‘The magnitude of the line element A7_ is very small. Its square i.e., A/? will be even smaller and can therefore be neglected in comparison to other terms in the equation. arta At or oral =r?-7? or Foal = 3 0*-)= 5 ene- 1 =3 (2r)(r -r) = rr’ -r) BG)ae lk & ects 2 Gye From Eqn. (1), B(r).al= ae a n= Fee a-n= ELECTROSTATIC POTENTIAL AND GAUSS'S THEOREM 81 be Perrerr] © 4ne9 It is clear from here that each elementary contribution to the line integral is the difference of two similar terms corresponding to the two ends of the segment under consideration. So, if we add up all the terms, the intermediate contributions cancel in pairs and we are left only with terms from the end points A and B. fp ec) 7-4 (41 Jn Br) db = Ge iy We conclude that the line integral does not depend upon the curve L. We may choose any path between A and B. The line integral of the electrostatic field would be still the same. In the above treatment, the line integral of the field has been evaluated for the electrostatic field of a single point charge. But the above treatment is equally valid for an electrostatic field produced by an assembly of point charges and continuous charge distributions. This argument follows from the principle of superposition. It must be clearly noted that it is the line integral of the electrostatic field which is path- independent. If the electric field is produced by moving charges, then the line integral would be path- dependent. In other words, the line integral of the time-dependent electric field depends upon path. , ‘Sreciat Case Fig. 2.3. Line integral of E If the electrostatic field is due to a single point charge q located at a general point P (a point other than origin), then the positions of A and B with reference to P are given by(r4 ~r, )and (Ty 7) respectively. Here, 7, is the position vector of P with respect to origin O. In-ml Im-n . aa 2 @ foto In this case, [LEG - dl = Ga, { =—s7 | ELECTROSTATIC FIELD—A CONSERVATIVE FIELD Let us consider a closed curve in space and evaluate the line integral of E over the closed curve (Fig. 2.4). Inthiscase, — f E(r). di = jaca. jac, ai oumel slang L! ‘The circle on the integral sign indicates a closed curve. i, BG). di = & (2-2), 4 (2-2) 4: TT. COMPREHENSIVE PHYSICS—xII So, the line integral of E over a closed curve is zero, Qs Since the line integral of electric field over aclosed curve is zero therefore the elec- trostatic field is a conservative field. In class XI, we came across another conservative field. This field is gravitational field. When a body is lifted, the amount of work done against the gravitational force does not depend upon the path along which the body is moved. It merely depends upon the initial and final Fig. 2.4. Line integral over closed curve Electrostatic field is a conservative field. So, work done in moving a test charge between any two points in an electric field is independent of the path between the two points, Also, the work done in moving a test charge over a closed path in an electric field is zero. PHYSICAL SIGNIFICANCE OF LINE INTEGRAL OF ELECTRIC FIELD Line integral of electric field between two points represents the work done by electric field in moving a unit positive charge between two points in the field. Work done in displacing unit positive charge through di = E . di. The electricfield E is assumed to be constant over di. By, Work done by electric field in moving unit positive charge from AtoB= |, B. di. Ifthe unit positive charge is to be moved against the direction of electric field, then work done A, fig CONCEPT OF ELECTROSTATIC POTENTIAL Let us recall the following two equations from the previous sections. q 4ne, paGy.a- 2 a og and jac d= i Rewriting these equations, we get Bla JBG).de- 4,4 4 ELECTROSTATIC POTENTIAL AND GAUSS'S THEOREM apts a and -d=- Br). dl =~ 7 (i cleemasibbexrscinoppniearlil ce halen conaomesinibe difference of two terms, In addition to source charge, these terms depend upon 7. So, both these equations can be combined to give us the following equation : By 4 J. 8G). d --vey+vey ) Here, V (r,) and V(r,) are scalar functions which depend upon the end points r, and r,. ‘The scalar function V(r )is the electrostatic potential at 7. Since the difference implies ‘final value — initial value’ therefore the difference of electro- static potentials is (V, — V,). Bcsast 0a FromEqn.(1), Virg)-Vory)=— J, BCP). dd (2) This equation is a basic defining equation which uniquely determines the electrostatic potential difference between two points. The electrostatic potential difference between two points is the negative of the line integral of the electrostatic field between the first and second points along any arbitrary path. Itis clear from the above discussion that ‘potential ata point’ is not uniquely defined. However, the ‘potential at point’is of great practical importance and there is a need to establish a formula for this. In order to do so, we use a simple technique. One of the two points is supposed to be at infinity and furthermore, the potential at this point; ipposed to be zero. Ifthe point Ais taken at infinity and the value ofthe function Vr,)is assumed tobe nro at infinity, Bi, + Virg) =- L E(r).dl {from Eqn. (2)} o Vop= 8G). a Dropping the subscript B, weget Vir)= | B(7). di In words, the electrostatic potential at any *point 7- is the line integral of the electrostatic field from the point r to infinity along any arbitrary path. Note, For the sake of simplicity, Vir,) and Vir) may be represented as V, and V, respectively. ere eta rit Form Yo- V,3 = je -— Unitsused: : Os asics “The term ‘point r” implies a point whose position with reference to the origin of the three-dimen- sional coordinate system is given by position vector 7. COMPREHENSIVE PHYSICS—xXiI 5000 r WM Example 2.01. The electric field outside a charged long straight wire is given by E =. Vm and is radially inward. What is the sign of the charge on the wire ? Find the value of Vp~V,ifrg=0.6 mand ry = 0.3 m. Which point is at the higher potential ? Solution. Since the field is directed towards the wire therefore we can safely conclude that the wire is charged negatively. Again, when we move from A to B, we are moving opposite to the direction of the field. So, B is at a higher potential than A. 6 5000 6 Va-Va=- K -aS dras s000 | wer[ = 5000 flog, 0.6 log, 0.81 = 6000 log, OF = 5000 tog, 2 = 5000 x 0.6931 V = 3465.5V 1 Example 2.02. A test charge q, is moved without acceleration from A to Cover the path shown in Fig. 2.5. Calculate the potential difference between A.and C. Solution. V,~V, Bias 4 J, E - di | cos 135° = cos (180°— 45") —»———__________» ~ B =~ cos 45” ‘B dt 1 =- fe cos 135° 5 “ha oD) But fia-an A, d a rr ad ABs Tyg = V2d From Eqn.(1), Vy-V,= & x (3 d=Ed Also, sin 45° For all points on the line BC, E is perpendicular to di . =Ve=0 or Vy=Vo Thus, potential diffrence between A and Cis the same as that between A and B, ie., Ed. Alternative Method Since work done is independent of path therefore we may directly move from A to C. Vo-Va=~ =-E-) fe [eos 180° =- 1] -Ed 4 [- fra-q] a You have either reached 2 page thts unevalale fer vowing or reached your ievina tit for his book. a You have either reached 2 page thts unevalale fer vowing or reached your ievina tit for his book. a You have either reached 2 page thts unevalale fer vowing or reached your ievina tit for his book. a You have either reached 2 page thts unevalale fer vowing or reached your ievina tit for his book. a You have either reached 2 page thts unevalale fer vowing or reached your ievina tit for his book. COMPREHENSIVE PHYSICS—xXil @ Example 2.08. The electric field at a point due toa point charge is 20 N C~' and the electric potential at that point is 10.J C!. Calculate the distance of the point from the charge and the magnitude of the charge. [CBSE 1996] Solution. E=20NC7,V=10NC+,r=?,q=? = ee i) E ime 20 i) and Ve =10 Ai) 10 Dividing (ii) by @), we get r= 55 m=0.5m ‘Again, from equation (ii), we get 9x10 x 2 =10 or q= x5C=5.56x10°C ¢ 4. Calculate the potential at a distance of 0.10 m from a proton in vacuum. The charge of proton is 9x10® positive and is equal to 1.6 x 10" coulomb. Tans, 1.44 x 10 V) 5. The electric potential at 0.2 m from a point charge is 50 V. Determine the magnitude and sign of the charge. (Ans. + 1.1 nC) 6. Ata point due to a point charge, the values of electric field intensity and potential are 30NC~ and 153 C* respectively. Calculate (i) magnitude of the charge, and (ii) distance of the charge from the point of observation. [CBSE 2002] (Ans. (i) 0.83 nC (ii) 0.5 m] 7. The electric potential at 0.1 m from a point charge is 50 V. What is the magnitude of the charge? (PSEB 1998 8; HSEB 1992] [Ams, 5.55 107°C} & Calculate the potential at a distance of 1 m from a proton in vacuum. The charge of proton is positive and is equal to 1.6 x 10-8 coulomb. (Ans. 1.44 x 10 V] ‘The electrostatic potential at 0.1 m from a point charge is 50 V. Determine the magnitude and sign ofthe charge. (HSEB 1992] (Ans. + 0.55 nC) x 10° 19 = 2116 x10" 2 1.44 « 104 volt 0.10 3x10° 3x108 x m 910° xq r 50x02 = prior Cm hARC 5x0. 3x10" C= 0.83 nC r = Ane, Vr PRINCIPLE OF SUPERPOSITION FOR ELECTROSTATIC POTENTIALS Consider two point charges +9, and + q,. Select any two points A and B in the combined electrostatic field of two charges 9, and q,. Choose arbitrarily any curve L between A and B. Weknow that V,-V,=~ fy B.di But & =i, + #, (Principle of superposition of electric fields) Va-Vae-f, @,+8).di 5 5. ge --fid-Pea So, electrostatic potential differences are additive. IfA is taken to infinity and subscript B is dropped, then foo v-[-[e.-di]+[-f'e-ai] Fig. 2.13. Principle of So, the electrostatic potential at B is the scalar sum of (i) po- tential at B due to charge q, alone, and (ii) potential at B due to charge q, alone. This result can be generalised for any number of charges. (2REBME ELECTRIC POTENTIAL AT A POINT DUE TO A NUMBER OF CHARGES Consider a number of point charges q,, 9» gy «= distances r,, ry Py)... 7, Respectively from jan observation; P. The resultant potential at P due to all the charges is the alge- braic sum of the potentials at P due to individual charges. This is because electric potential is a scalar quantity. The resultant po- tential at Pis given by BM 8 Gi toe 8 ney 7 © 4nty le ry" Ang Ty Fig. 2.14. Potential due to number of point charges 1 Example 2.09. Two point charges 4 C and - 2 uC are separated by a distance of 1 min air. At what point on the line joining the charges is the electric potential zero ? [CBSE 2001] —_+.—____. Solution, IfN is the required point, then te i Se 1 4x10 1 2x10% ot Gey oz trey i-x 7° 7 I-x Fig. 2.15, COMPREHENSIVE PHYSICS—) 42 x I-z or 4~4r=2r or Gx=4 4 2 or x= m= 3 m=0667m < Mt Example 2.10. Two charges 5 x 10% Cand-3 x 10 Care located 0.16 m apart. At what point on the line joining the two charges is the electric potential zero ? Take the potential at infinity to be zero. Solution. Let the potential be zero at a point P whose distance is x metre from the charge q, =5 x 10 C. The distance of P from q, =~3 x 10 Cis (0.16 -x) metre. Potential at P due toq, = —— 2 a $ % ance = a Panter aietngy=— gh GBs But total potential at P should be zero, a er.) tne, O16-x qe _3x10% a 5x10% or &=0.8 or x=O0.1m 4 In this problem, it would be interesting to find another point where potential is zero. This point is possibly to the right of negative charge. If the distance of this point from the positive charge is x metre, then its distance from the negative charge is @- 0.16) m. Now, #-016 _3x10% = =04 =04 Feats OF R=08m or x=04m WW Example 2.11. Calculate the potential at the centre O of the square shown in Fig. 2.17. (HSEB 1998) Solution. Let us first calculate the distance (OB = OC = OA = OD) of each charge from O. BD?=1?+1?=2orBD= 2m 1 1 0B(-0C=0A=0D)= 5 BD= Jy m=0.707m Potential at O due to charges at A, B, C and D is given by 1 (-2+3+2)10%C Y= Gre 0.707 m 4x10%C _ 360 = Nm? Cc =. =9x 10°Nm*C* a = oao7 v=500.2V 4 a You have either reached 2 page thts unevalale fer vowing or reached your ievina tit for his book. a You have either reached 2 page thts unevalale fer vowing or reached your ievina tit for his book. a You have either reached 2 page thts unevalale fer vowing or reached your ievina tit for his book. COMPREHENSIVE PHYSICS—Xil Fig. 2.24 Fig. 2.25 16. Fig. 2.25 shows a charge array known as an electric quadrupole. For a point on the axis of the quadrupole, obtain the dependence of potential on r for ~ >> 1, Contrast your result with that due to an electric dipole and an electric monopole (ie., a single charge). [Ans. Ur; Ur? ; Ur] 17. Two charges are placed 0.12 m apart. The magnitude of charges are 60 x 10-° C and ~ 40 x 10°C. Find the potential (a) midway between the charges ; (b) at a point 0.04 m to the right of ~ 40 x 10°C charge. Use B= 4 and V= r 8x10 40x10% 5 x10% = 9x10%] 8X10" _ 40x10" v<8 wf 01 02° On or V=9 x 10[80- 200 + 50] volt =~ 6300 volt v or 22 9x10" x10 v= 2210 10% =2.7 x 108 volt 9x10" V=—Jgq 5-12 + 11-10+9-8+7-6) [2-2-3 +6} volt x 10 volt =-18«10V 4 (Ans. (a) 3 x 103 V (6) ~ 5625 V} ‘The magnitudes of the three field vectors are the same. Moreover, they are inclined to each other at 120° ve-g-{4-2,1] 4ney|7-a rr r+aj a © Ame rir? a") Neglecting a? in comparison to r?, y= 2408 Arey” 9x 10° x10-° @V = jog [60 - 401 V =3x10°V 60 40 Vv=9x 108 x 19-9 | 2-40 (b) V = 9 x 10° x 10° is 4] = 9[375 - 1000] volt = - 5625 volt a You have either reached 2 page thts unevalale fer vowing or reached your ievina tit for his book. (iii) The electric potential at + due to volume charge distribution is given by , where 7’ is the position vector of the element du. ated WM Example 2.17. Calculate the potential at a point on the axis of a ring of charge of radius a. Given : Charge on the ring =q and distance of the observation point from the centre of the ring is x. Solution. The potential at the observation point P 7 1 r due to an elementary charge dq is given by dV = Fee 4 cera mee = ae 7, ter ZS But r= yetee? 3. Ve gee co ar) f+ 2 POTENTIAL DUE TO A CHARGED CONDUCTOR (HOLLOW OR SOLID) Case I. At an external point The charge Q on the sphere behaves as if concentrated at the centre. Case II. At a point on the surface Now, r=R Case IIL. At a point inside the conductor We know that n-- But elotre field ata point inside the conductor in sar. r=R — on & Fig. 2.28 °~ dr = V=constant [-. Differential coefficient of a constant quantity is zero.) Potential at every point inside the conductor is the same and equal to potential at the surface. a You have either reached 2 page thts unevalale fer vowing or reached your ievina tit for his book. a You have either reached 2 page thts unevalale fer vowing or reached your ievina tit for his book. ELECTROSTATIC POTENTIAL AND GAUSS'S THEOREM 101 W Example 2.21. Bight charged water droplets, each with a radius of 1 mm and a charge of 10-1” C, coalesce to form a single drop. Calculate the potential of the bigger drop. Solution. Let R and r be the radii of the bigger drop and one droplet respectively. Ve of 8 droplets = Voh of bigger droy lume o ume ? Note that the concept of capac: 8x4 ata 4 eR orR= (ram ‘has not been used in this solution. or R=2x1x10%m=2x 109m t Charge on bigger drop, Q=8x 101°C 1mm] " 1 @Q 8x 107 Potential ofthe bigger drop = 2 Ft = 9x10 5, io volt = 3600 volt EXERCISE 19. Twenty seven charged water droplets, each with a radius of 1 x 10’ m and a charge of 10- C, coalesce to form a single drop. Calculate the potential of the bigger drop. (Ans. 81 V} ° 12 [Hine $ xa =27 $8, Rear: V= 1_ @ _ 9x10? x 27x10 ier E Tage velt= 81 vat] RELATION BETWEEN POTENTIAL GRADIENT AND ELECTRIC FIELD STRENGTH Consider two points A and B in the electrostatic field 2 of point charge +q placed atO.LetussssumethatAand 9 reer > Bare soclose that the electricfield intensity betweenAand “a se Bis uniform and is equal to E. a _, Inorder to move a test charge q, from A to B, a force Fig. 2.31, Relation between ‘ 4S 0 Potential at Pdue to electricdipole, V= =~ —2 Tq sJiiofia Ly 1 (ee="] 1 qx 1 op i al=pl “ qxQl=p * Gre, 7 | PF |> Ifr >>, then ELECTRIC POTENTIAL DUE TO AN ELECTRIC DIPOLE AT A POINT ON THE EQUATORIAL LINE Potential at P due to charge += 1g 4ne, BP 1 + 0 AP Potential at P due to charge~q =- a You have either reached 2 page thts unevalale fer vowing or reached your ievina tit for his book. [ ELECTROSTATIC POTENTIAL AND GAUSS'S THEOREM 107 va an rar (1-2) re (1-2 xo) r nore Applying Binomial theorem and neglecting squares and higher powers, we get dad {1+4 cose) wf) Ror\ or Again, ry? =r? + [2 2rl cos (180-8) or 7,2 =r? + 1? + 2rl cos 8 2 oe)” P or myer (1+ cose) [Nese 4 7 -va o Jet (1+2 cose} mork or Applying Binomial theorem and neglecting squares and higher powers, we get 21 (1-4 cose) (8) mor\ 7 ca ais h cosets! Combining (1), (2) and (3), we get V = trey [i+4 cose 1+ cos] qx2cos@ 1 peose or Ve eg OV" Gee oe where p is the electric dipole moment of the electric dipole. ‘Two Grecia, Cases Case I. When the observation pointis on the axial fine, then @ = 0° 12 V* Greg 7? Case II. When the observation point is on the equatorial line, then 8 = 90°. So, V=0. An Alternative Approach AP = A’P = OP +0OA’ = OP + AO cos8 =r+1lcos® Similarly, BP « B’P = OP - OB’ =r-/ cos 6 1 Pac Fete 8° “Gar sii i _@ a w "ane, r—1c08 0 Iv peqx2ll Potential at P due to charge ‘+ q’, V, = —— Net potential at P, V=V,+V, ot j-_ 2 _,__¢_|__ 2/2 _..__ 4 _ 4mey | r+lcos8 r—lcos@| 4ne,|r—lcos@ r+icos® a [SS -(r- see 1 qx2lcos® 1 peed = Gre, =F eos® @ Frey 7? =F cos? @ ~ 4neg r?-T? cos® 6 ee 108 COMPREHENSIVE PHYSICS—XII ‘When P lies on the axial line of the dipole, @ = 0°. cos 6 = cos 0° = - P Vv Ane, (r* Ift < ada . Again, inthe absence of A. B % fa & chai » the potential energy ea system of charges q, and a ee se Fig. 2.44. Electrostatic P.E. of a 3 system of three charges 4meq 73 Since energy is a scalar quantity therefore the total energy of the system can be obtained by adding up the energies of every pair of charges. Ja, 12% +4) Tas ns Aliter. (In terms of position vectors) Let us determine the electric potential energy of a system of three charges q,, q, and gs located at 7%, rz and r, respectively. To begin with, let us assume that all the three charges are at infinity. Work done in bringing charge q, from infinity to its location a is zero, Work done in bringing charge g, from infinity to its location 7, is given by 1 a Wi= are, in ra -Al Let W,, be the work done in bringing charge q, from infinity to r, in the electric field of charge q,. Let W,, be the work done in bringing charge q, from infinity to 7, in the electric field of charge q,. - 4% wy __1 4% i 0 Ir -rs| ‘Total work done in bringing the charges of the system from infinity to their respective positions is given by W=W,, + Wy; + Wey ‘This work done is stored as electrostatic potential energy U. 1 N92. 19s, Gods tm, | 2 2 t> 3 tS ° \lnm-al la-al Ian

Você também pode gostar